Exam 4 Test Banks Flashcards

1
Q

The nurse is teaching a client about side effects and adverse reactions of a PDE5 inhibitor. What information does the nurse include? (Select all that apply.)

a. Refrain from eating citrus fruit within 24 hours of taking the medication.
b. Stop using this drug if your primary health care provider prescribes a nitrate.
c. Do not drink alcohol before having sexual intercourse.
d. Muscle cramps, nausea, and vomiting are possible if you take more than 1 pill a day.
e. Take this medication within 30 to 60 minutes of having sexual intercourse.
f. Change positions slowly especially if you also take an anti-hypertensive drug.

A

b. Stop using this drug if your primary health care provider prescribes a nitrate.
c. Do not drink alcohol before having sexual intercourse.
d. Muscle cramps, nausea, and vomiting are possible if you take more than 1 pill a day.
f. Change positions slowly especially if you also take an anti-hypertensive drug.

Rationale: A PDE5 inhibitor is used to treat erectile dysfunction. The client should avoid grapefruit or grapefruit juice while taking these drugs.
Taking a PDE5 inhibitor along with a nitrate can cause a profound drop in blood pressure.
Alcohol may interfere with the ability to have an erection. Muscle cramps, nausea, and vomiting are possible side effects if more than 1 pill a day are taken.
Each medication has its own directions for how soon to take it before intercourse, from 15 minutes to 2 hours.
Any PDE5 drug can lower blood pressure so the nurse alerts the client of safety precautions.

How well did you know this?
1
Not at all
2
3
4
5
Perfectly
2
Q

A client came to the clinic with erectile dysfunction. What are some possible causes of this condition that the nurse could discuss with the client during history taking? (Select all that apply.)

a. Recent prostatectomy
b. Long-term hypertension
c. Diabetes mellitus
d. Hour-long exercise sessions
e. Consumption of beer each night
f. Taking long hot baths

A

a. Recent prostatectomy
b. Long-term hypertension
c. Diabetes mellitus
e. Consumption of beer each night

Rationale: Organic erectile dysfunction can be caused by surgical procedures, vascular diseases such as hypertension and its treatment, diabetes mellitus, and alcohol consumption. There is no evidence that exercise or hot baths are related to this problem.

How well did you know this?
1
Not at all
2
3
4
5
Perfectly
3
Q

A nurse is reviewing concepts related to physiological responses that occur during sexual acts. Which statement should the nurse identify as not being accurate?

a. During resolution, ADH and oxytocin are released.
b. Most often in males, orgasm occurs with ejaculation.
c. Genital congestion occurs as part of a reflexive response.
d. Dopamine secretion acts as an inhibitory transmitter.

A

d. Dopamine secretion acts as an inhibitory transmitter.

Rationale: The general phases of sexual arousal include motivation, arousal, genital congestion, orgasm and resolution. Dopamine secretion is considered to be an excitatory and released during the arousal stage.
Orgasm and ejaculation occur more frequently in males.
Genital congestion is under reflexive autonomic response.

How well did you know this?
1
Not at all
2
3
4
5
Perfectly
4
Q

The nurse is obtaining a sexual history from an adolescent patient. Which finding has the greatest implication for this patient’s care?

a. Patient denies any sexual activity.
b. Patient states that he/she uses “safe sex” practices.
c. Patient states that he/she is in a monogamous relationship.
d. Patient has been intimate with more than one person in the last year.

A

d. Patient has been intimate with more than one person in the last year.

Rationale: The Center for Disease Control (CDC) had identified the 5P’s with regard to obtaining information for a sexual history.
They focus on partners, practices, protection from infection, past history of infection, and prevention of pregnancy.
An individual who has had more than one partner within the time frame should be questioned regarding condom use.
Denial of sexual activity is part of the patient’s self-disclosure. The patient stating that he/she is in a monogamous relationship again represents self-disclosure.
Use of “safe sex” practices may need to be further explored but it does not have the greatest implication at this point.

How well did you know this?
1
Not at all
2
3
4
5
Perfectly
5
Q

A 45-year-old female patient, gravida 3 para 3, presents with complaints of decreased desire to engage in sexual activity with her husband as it is becoming more painful. What physical assessment data should the nurse focus on?

a. Urine culture to identify potential STD.
b. Obtain vital signs as a baseline to rule out infection.
c. Prepare for a vaginal exam.
d. Inspection of the abdomen for pelvic mass

A

c. Prepare for a vaginal exam.

Rationale: Based on the patient’s reported complaint and obstetrical history, it may be likely that the patient has a pelvic prolapse.
Therefore, a vaginal exam would be indicated to help identify possible anatomical changes.
There is no clinical data that supports a potential pelvic mass and inspection alone would not confirm this finding.
Obtaining vital signs as well as a urine culture may
be needed, but the focus should be on determination of physical findings related to the pelvic area.

How well did you know this?
1
Not at all
2
3
4
5
Perfectly
6
Q

An adult patient comes for a well-check up to the primary care provider’s office. In completing the office admission form, the
patient does not indicate gender on the form and seems somewhat agitated when providing the form back to the nurse. How should the nurse respond?

a. Ask the patient to complete all of the information at this time.
b. Ask the patient if you can assist with completing the form.
c. The nurse should just indicate which gender she/he thinks is appropriate.
d. Tell the patient that if the form is not completed, then the doctor will not see you.

A

b. Ask the patient if you can assist with completing the form.

Rationale: Gender identity is defined by the individual patient. The nurse should not designate this description or identity nor should the nurse tell the patient that if the form is not completed, that the patient will not be seen by the healthcare provider.
Asking the patient to complete the information without acknowledging that the patient is exhibiting signs of distress is not therapeutic.
The nurse should offer to provide assistance to the patient.

How well did you know this?
1
Not at all
2
3
4
5
Perfectly
7
Q

A nurse is working with a male patient being treated for erectile dysfunction. Which statement indicates that additional teaching is needed?

a. “I like to go walking around my community each night after dinner.”
b. “I have a few drinks during the week when I go out after work.”
c. “I have maintained my weight for the past 5 years after losing 20 pounds.”
d. “I monitor my blood pressure at home using a portable cuff.”

A

b. “I have a few drinks during the week when I go out after work.”

Rationale: Erectile dysfunction (ED) is a common problem affecting the male population and can be chronic or transient in nature.
Alcohol use can affect ED, so the patient’s reported alcohol intake indicates that additional teaching is warranted.
Exercise, maintaining a healthy weight, and monitoring of blood pressure are examples of appropriate activities.

How well did you know this?
1
Not at all
2
3
4
5
Perfectly
8
Q

A nursery nurse performing the first physical assessment of the newborn observes that there is no clear identification of genitalia as being either female or male. How should the nurse identify this newborn?

a. Gender neutral
b. Bisexual
c. Observation of intersex
d. Asexual

A

c. Observation of intersex

Rationale: Intersex represents a group of conditions where the external genitalia of an infant does not appear as either male or female and/or is not consistent with genetic sex or organs. The nurse cannot attribute sexual preference such as asexual or bisexual.
Gender neutral does not apply to this clinical situation.

How well did you know this?
1
Not at all
2
3
4
5
Perfectly
9
Q

A nurse working with a family whose child has recently told them that he identifies with the LGBTQ community asks the nurse to explain how this happened considering the fact that the child was raised as a male and played with appropriate toys. What is the best nursing response to the family’s concerns?

a. Tell them that there is no need for concern for their child has shared this information with them.
b. Ask the parents if they ever noticed something different about their son as he was growing up.
c. Explain that sexual orientation changes can occur over time.
d. Suggest that this behavior may be temporary.

A

c. Explain that sexual orientation changes can occur over time.

Rationale: Sexual orientation and gender identification is now thought of as a fluid concept, with the term sexual fluidity being used to convey this meaning for individuals who identify with other than heterosexual relationships.
The nurse should respond to the parent’s concerns and not minimize their reaction but rather let them know that it is the chosen response of their child.
Relating sexual orientation or gender identification to how one was raised indicates an implied bias. Telling the family that the behavior may be temporary is not correct.

How well did you know this?
1
Not at all
2
3
4
5
Perfectly
10
Q

A nurse is assessing a client who presents with a scaly rash over the palms and soles of the feet and the feeling of muscle aches and malaise. Which action by the nurse is most appropriate?

a. Reassure the client that these lesions are not infectious.
b. Assess the client for hearing loss and generalized weakness.
c. Don gloves and further assess the client’s lesions.
d. Take a history regarding any cardiovascular symptoms.

A

c. Don gloves and further assess the client’s lesions.

Rationale: The client is displaying symptoms similar to secondary syphilis, with flu-like symptoms and rash due to the spirochetes circulating throughout the bloodstream. Therefore, the nurse needs to further assess the client’s lesions with gloves since the client is highly contagious at this stage.
Tertiary syphilis may display in the form of cardiovascular or central nervous system symptoms.
Neurosyphilis can appear at any time, in any state, and can include hearing loss.

How well did you know this?
1
Not at all
2
3
4
5
Perfectly
11
Q

A male client is diagnosed with primary syphilis. Which question by the nurse is a priority at this time?

a. “Have you been using latex condoms?”
b. “Are you allergic to penicillin?”
c. “When was your last sexual encounter?”
d. “Do you have a history of sexually transmitted infections?”

A

b. “Are you allergic to penicillin?”

Rationale: Benzathine penicillin G is the evidence-based treatment for primary, secondary, and early latent syphilis.
The client needs to be assessed for allergies before treatment. The other questions would be helpful in the client’s history of sexually transmitted infections but not as important as knowing whether the client is allergic to penicillin.

How well did you know this?
1
Not at all
2
3
4
5
Perfectly
12
Q

A client with genital herpes has painful blisters on her vulva. After teaching the client self-care measures, which statement
indicates the need for further education?

a. “Pouring water over my genitals will decrease the pain of urinating.”
b. “I will wash my hands carefully after applying ointment.”
c. “When I don’t have lesions, I am not contagious to my sexual partner.”
d. “I should increase my fluid intake when I have open lesions.”

A

c. “When I don’t have lesions, I am not contagious to my sexual partner.”

Rationale: A client with genital herpes can still spread the disease when asymptomatic through viral shedding.
The client is taught to use condoms with all sexual activity. Pouring water over the genitals (or urinating in the shower) will help decrease the pain of urine passing over open lesions.
Good handwashing is important.
Open lesions can lead to fluid loss so the client is taught to increase fluid intake.

How well did you know this?
1
Not at all
2
3
4
5
Perfectly
13
Q

A 30-year-old male client is asking the nurse about the vaccine for human papilloma virus (HPV). Which statement by the nurse is accurate?

a. “Gardasil protects against all HPV strains.”
b. “You are too old to receive the vaccine.”
c. “Only females can receive the vaccine.”
d. “You will only need 1 dose of the vaccine.”

A

d. “You will only need 1 dose of the vaccine.”

Rationale: Gardasil is used to provide immunity for HPV types 6, 11, 16, and 18 and Gardasil 9 protects against 5 more strains.
The vaccine is recommended for people aged 9 to 26 years of age, but Gardasil 9 can be given up to age 45.
Both males and females can get the vaccine.
Depending on the timing and type of vaccine, either 2 to 3 doses are required.

How well did you know this?
1
Not at all
2
3
4
5
Perfectly
14
Q

A client with multiple sexual partners has been assessed for symptoms of dysuria and green, malodorous vaginal discharge. The nurse administers an injection of ceftriaxone and gives the client a prescription for doxycycline. The client asks why two drugs are needed. What answer by the nurse is best?

a. “It is very common to be infected with both gonorrhea and chlamydia.”
b. “Giving two medications increases the chance of curing the infection.”
c. “Some people are not affected by the injection and need more medication.”
d. “This will prevent you from needing a 3-month follow-up test.”

A

a. “It is very common to be infected with both gonorrhea and chlamydia.”

Rationale: This client has signs of gonorrhea. Co-infection with gonorrhea and chlamydia is common, so the client being treated for gonorrhea also needs treatment for chlamydia with oral antibiotics.
It is fairly accurate to say two medications increases the chance of cure, but does not really explain the situation.
Giving the client two medications is not because some people are not affected by the injection nor is it to prevent needing a 3-month follow-up test.
Testing for re-infection with chlamydia is recommended by the
CDC.

How well did you know this?
1
Not at all
2
3
4
5
Perfectly
15
Q

While evaluating a client for treatment of gonorrhea, which question is the most important for the nurse to ask?

a. “Do you have a history of sexually transmitted infection?”
b. “When was your last sexual encounter?”
c. “When did your symptoms begin?”
d. “Can you remember your partners and contact them to get treated?”

A

d. “Can you remember your partners and contact them to get treated?”

Rationale: Sexual partners, as well as the client, should be tested and treated for gonorrhea.
Asking about sexually transmitted infection history, last sexual encounter, and onset of symptoms would be helpful with the history taking, but the priority is treating the client’s sexual partners to limit the spread of the infection.

How well did you know this?
1
Not at all
2
3
4
5
Perfectly
16
Q

A client has been treated for syphilis with IM penicillin. The next day the client calls the clinic to report fever, chills, achy muscles, and a worsening rash. What statement by the nurse is most appropriate?

a. “You must be allergic to penicillin; over-the-counter antihistamines will help.”
b. “Please go to the nearest emergency department if you develop shortness of breath.”
c. “You can take acetaminophen or ibuprofen for the pain and achiness.”
d. “I think you should come into the clinic either today or tomorrow and be checked.”

A

c. “You can take acetaminophen or ibuprofen for the pain and achiness.”

Rationale: This client has signs of a Jarisch-Herxheimer reaction which is caused when the organisms’ cell walls are disrupted and cellular contents are released rapidly.
It is usually self-limiting and benign. Antipyretics and mild analgesics treat the symptoms.
The client does not need to monitor for shortness of breath, come into the clinic, or get antihistamines for an allergic reaction.

How well did you know this?
1
Not at all
2
3
4
5
Perfectly
17
Q

A 24-year-old female has been diagnosed with genital warts. Which action by the nurse is best?

a. Encourage the client to complete STI screening.
b. Recommend an over-the-counter wart treatment for genital tissue.
c. Report the case to the Centers for Infection Control and Prevention (CDC).
d. Discuss popular options for contraception.

A

a. Encourage the client to complete STI screening.

Rationale: Clients with HPV should be fully screened for other STIs since co-infection is common.
Over-the-counter treatments should not be applied to genital tissue.
HPV is not reportable.
Contraception is not related.

How well did you know this?
1
Not at all
2
3
4
5
Perfectly
18
Q

A female client returned to the clinic with a yellow vaginal discharge after being treated for a Chlamydia infection 3 weeks ago. Which statement by the client alerts the nurse that there may be a recurrence of the infection?

a. “I did practice abstinence while taking the medication.”
b. “I took doxycycline two times a day for a week.”
c. “I never told my boyfriend about the infection.”
d. “I did drink wine when taking the medication for Chlamydia.”

A

c. “I never told my boyfriend about the infection.”

Rationale: There is a good possibility that the boyfriend reinfected the client after the medication regimen was finished.
Both the client and the boyfriend need to be treated.
The other statements were in compliance with the recommendations of abstinence and the usual
medication regimen with doxycycline.
Wine should not interfere with the treatment.

How well did you know this?
1
Not at all
2
3
4
5
Perfectly
19
Q

A college student seeks information from the school’s nurse about how to avoid sexually transmitted infections (STIs) without abstinence as a choice. Which statement by the nurse is best?

a. “Urinating after intercourse will eliminate the risk of infection.”
b. “A vaccine can prevent genital warts caused by some strains of the human papilloma virus (HPV).”
c. “Oral contraception can prevent pregnancy and STIs.”
d. “Good handwashing helps prevent infection associated with STIs.”

A

b. “A vaccine can prevent genital warts caused by some strains of the human

Rationale: Gardasil and Gardasil 9 are used to provide immunity for HPV types 6, 11, 16, and 18 and others that are at high risk for cervical cancer and genital warts.
While there is some truth that urination after intercourse may decrease the risk of infection by flushing out organisms, it does not eliminate the risk of contaminating bacteria traveling up the urethra or from skin-to-skin contact.
The other statements are not accurate.

How well did you know this?
1
Not at all
2
3
4
5
Perfectly
20
Q

A client has a positive HSV-2 test but is asymptomatic. What action by the nurse is best?

a. Encourage the client to have frequent STI screening.
b. Teach the client ways to prevent getting STIs.
c. Provide the same education as if the client were symptomatic.
d. Inform the client that partner notification is unnecessary.

A

b. Teach the client ways to prevent getting STIs.

How well did you know this?
1
Not at all
2
3
4
5
Perfectly
21
Q

A primary care clinic sees some clients with sexually transmitted infections. Which diseases would the nurse be required to report to the local authority? (Select all that apply.)

a. Chlamydia
b. Gonorrhea
c. Syphilis
d. Human immune deficiency virus
e. Pelvic inflammatory disease
f. Human papilloma virus

A

a. Chlamydia
b. Gonorrhea
c. Syphilis
d. Human immune deficiency virus

Rationale: Chlamydia, gonorrhea, syphilis, human immune deficiency virus (HIV), and acquired immune deficiency syndrome (AIDS) are all reportable to local authorities in every state.
Pelvic inflammatory disease and HPV do not need to be reported.

How well did you know this?
1
Not at all
2
3
4
5
Perfectly
22
Q

A nurse wants to reduce the risk potential for transmission of chlamydia and gonorrhea with a client diagnosed with both
infections. Which items should be included in the client’s teaching plan? (Select all that apply.)

a. Expedited partner therapy
b. Abstinence until therapy is completed
c. Use of intrauterine devices
d. Proper use of condoms
e. Rescreening for infection
f. Use of oral contraception

A

a. Expedited partner therapy
b. Abstinence until therapy is completed
d. Proper use of condoms
e. Rescreening for infection

Rationale: As part of client/partner education, the nurse should explain expedited partner therapy (the practice of treating both sexual partners by providing medication to the client for the partner).
The nurse should also emphasize the need for abstinence from sexual intercourse until treatment is finished, proper use of condoms, and rescreening.

How well did you know this?
1
Not at all
2
3
4
5
Perfectly
23
Q

A client being treated for syphilis visits the office with a possible allergic reaction to benzathine penicillin G. Which abnormal findings would the nurse expect to document? (Select all that apply.)

a. Red rash
b. Shortness of breath
c. Heart irregularity
d. Chest tightness
e. Anxiety
f. Confusion

A

a. Red rash
b. Shortness of breath
d. Chest tightness
e. Anxiety

Rationale: The nurse should keep all clients at the office for at least 30 minutes after the administration of benzathine penicillin G.
Allergic signs and symptoms consist of rash, shortness of breath, chest tightness, and anxiety.
Heart irregularity and confusion are not seen as allergic manifestation.

How well did you know this?
1
Not at all
2
3
4
5
Perfectly
24
Q

The nurse is teaching a client who is taking an oral antibiotic for the treatment of a sexually transmitted infection (STI). Which statements by the client indicate a correct understanding of the treatment? (Select all that apply.)

a. “I need to drink at least eight glasses of fluid each day with my antibiotic.”
b. “I should read the instructions to see if I can take the medication with food.”
c. “Antacids should not interfere with the effectiveness of the antibiotic.”
d. “I need to wait 7 days after this injection to engage in intercourse.”
e. “It should not matter if I skip a couple of doses of the antibiotic.”

A

a. “I need to drink at least eight glasses of fluid each day with my antibiotic.”
b. “I should read the instructions to see if I can take the medication with food.”
d. “I need to wait 7 days after this injection to engage in intercourse.”

Rationale: When a client is being treated with an oral antibiotic for an STI, 8 to 10 glasses of fluid should be routine, medication instructions should be reviewed, and at least a week break should occur between the antibiotic and sexual intercourse to allow for the medication’s full effects if the medication was given in a single dose.
Use of antacids and missing doses could decrease the
effectiveness of the antibiotic.

How well did you know this?
1
Not at all
2
3
4
5
Perfectly
25
Q

A nurse is reviewing the chart of a new client in the family medicine clinic and notes that the client is identified as “George Smith.” The nurse enters the room and finds a woman in a skirt. What action by the nurse is best?

a. Apologize and declare confusion about the client.
b. Ask Mrs. Smith where her husband is right now.
c. Ask the client about preferred forms of address.
d. Explain that the chart must contain an error.

A

c. Ask the client about preferred forms of address.

Rationale: The nurse may encounter transgender clients whose outward appearance does not match their demographic data. In this case, the nurse should greet the client and ask the client to explain his or her preferred forms of address.
Lengthy apologies can often create embarrassment.
The nurse should not assume that the client is not present in the room.
The chart may or may not contain errors, but that is not related to determining how the client prefers to be addressed.

How well did you know this?
1
Not at all
2
3
4
5
Perfectly
26
Q

A nurse is providing health teaching to a middle-aged male-to-female (MtF) client who has undergone gender-reaffirming surgery. What information is most important to this patient?

a. “Be sure to have an annual prostate examination.”
b. “Continue your normal health screenings.”
c. “Try to avoid being around people who are ill.”
d. “You should have an annual flu vaccination.”

A

a. “Be sure to have an annual prostate examination.”

Rationale: The MtF client retains the prostate, so annual screening examinations for prostate cancer remain important. The other statements are good general health teaching ideas for any patient.

How well did you know this?
1
Not at all
2
3
4
5
Perfectly
27
Q

A transgender client is taking transdermal estrogen. What assessment finding does the nurse report immediately to the primary health care provider?

a. Breast tenderness
b. Headaches
c. Red, swollen calf
d. Swollen ankles

A

c. Red, swollen calf

Rationale: A red, swollen calf could be a sign of a deep-vein thrombosis, a known adverse effect of estrogen.
The nurse reports this finding immediately.
The other signs and symptoms are also side effects of estrogen, but do not need to be reported as a priority.

How well did you know this?
1
Not at all
2
3
4
5
Perfectly
28
Q

A transgender client taking spironolactone is in the internal medicine clinic reporting heart palpitations. What action by the nurse takes priority?

a. Draw blood to test serum potassium.
b. Have the client lie down and rest.
c. Obtain a STAT electrocardiogram (ECG).
d. Take a set of vital signs.

A

c. Obtain a STAT electrocardiogram (ECG).

Rationale: Spironolactone is a potassium-sparing diuretic, and hyperkalemia can cause cardiac dysrhythmias.
The nurse’s priority is to obtain an ECG, and then to facilitate a serum potassium level being drawn.
Having the client lie down and obtaining vital signs are also
important care measures, but are not the most important at this time.

How well did you know this?
1
Not at all
2
3
4
5
Perfectly
29
Q

The nurse is teaching a transgender client about taking testosterone. What statement by the client indicates good understanding?

a. “My periods should stop immediately.”
b. “Some effects can take up to a year to see.”
c. “I am glad I don’t have to watch my diet.”
d. “There are very few side effects since it’s a normal hormone.”

A

b. “Some effects can take up to a year to see.”

Rationale: Testosterone is used as masculinizing drug therapy. Some desired effects may take up to a year to be noticed. Menses should stop within the first few months of therapy. Testosterone increases the risk of heart disease, so clients should follow a heart-healthy diet.
Testosterone has several side effects, including acne, seborrhea, weight gain, edema, headaches, and possible psychosis.

How well did you know this?
1
Not at all
2
3
4
5
Perfectly
30
Q

A client is preparing for MtF gender–affirming surgery. The client is worried about the voice not sounding feminine enough. What action by the nurse is best?

a. Ask if the client has considered vocal cord surgery to change the voice.
b. Refer the client for vocal therapy with a speech–language pathologist.
c. Teach the client that there will be no effect on the patient’s voice.
d. Tell the client that the use of hormones will eventually change the voice.

A

b. Refer the client for vocal therapy with a speech–language pathologist.

Rationale: Male-to-female clients can consult with a speech–language pathologist for vocal training to help with intonation and pitch.
While vocal surgery is possible, it may not be the best first option due to cost and invasiveness.
Telling the client that there will be no change to the voice does not give the client information to address the concern.
While the hormones this client is taking will not affect the voice, simply stating that fact does not help the client manage this issue.

How well did you know this?
1
Not at all
2
3
4
5
Perfectly
31
Q
  1. Which statement made by an adolescent girl indicates an understanding about the prevention of sexually transmitted diseases (STDs)?

a. “I know the only way to prevent STDs is to not be sexually active.”
b. “I practice safe sex because I wash myself right after sex.”
c. “I won’t get any kind of STD because I take the pill.”
d. “I only have sex if my boyfriend wears a condom.”

A

a. “I know the only way to prevent STDs is to not be sexually active.”

Rationale: Abstinence is the only foolproof way to prevent an STD. STDs are transmitted through body fluids (semen, vaginal fluids, blood).
Perineal hygiene will not prevent an STD.
Oral contraceptives do not protect women from contracting STDs.
A condom can reduce but not eliminate an individual’s chance of acquiring an STD.
However, the nurse should encourage condom use 100% of the time to decrease the risk.

How well did you know this?
1
Not at all
2
3
4
5
Perfectly
32
Q
  1. Which STD should the nurse suspect when an adolescent girl comes to the clinic because she has a vaginal discharge that is white with a fishy smell?

a. Human papillomavirus
b. Bacterial vaginosis
c. Trichomonas
d. Chlamydia

A

b. Bacterial vaginosis

Rationale: Bacterial vaginosis is characterized by a profuse, white, malodorous (fishy smelling) vaginal discharge that sticks to the vaginal walls.
Manifestations of the human papillomavirus are anogenital warts that begin as small papules and grow into clustered lesions.
Infections with Trichomonas are frequently asymptomatic. Symptoms in females may include dysuria, vaginal itching, burning, and a frothy, yellowish-green, foul-smelling discharge. Many people with chlamydial infection have few or no symptoms.
Urethritis with dysuria, urinary frequency, or mucopurulent discharge may indicate chlamydial infection.

How well did you know this?
1
Not at all
2
3
4
5
Perfectly
33
Q

The conscious decision on when to conceive or avoid pregnancy throughout the reproductive years is called

a. family planning.
b. birth control.
c. contraception.
d. assisted reproductive therapy.

A

a. family planning.

Rationale: Family planning is the process of deciding when and if to have children.
Birth control is the device and/or practice used to reduce the risk of conceiving or bearing children.
Contraception is the intentional prevention of pregnancy during sexual intercourse.
Assisted reproductive therapy is one of several possible treatments for infertility.

How well did you know this?
1
Not at all
2
3
4
5
Perfectly
34
Q

While instructing a couple regarding birth control, the nurse should be aware that the method called natural family planning

a. is the same as coitus interruptus, or “pulling out.”
b. uses the calendar method to align the woman’s cycle with sexual activity.
c. is used by 2% of Roman Catholics.
d. relies on barrier methods during fertility phases.

A

c. is used by 2% of Roman Catholics.

Rationale: Natural family planning is used by about 2% of Roman Catholics.
It is not the same a coitus interruptus. It uses a variety of methods to determine a woman’s fertility.
Those practicing natural family planning do not use barrier methods at any time.

How well did you know this?
1
Not at all
2
3
4
5
Perfectly
35
Q

A nurse is providing information about contraceptives to a couple. Which contraceptive method provides protection against sexually transmitted diseases?

a. Oral contraceptives
b. Tubal ligation
c. Male or female condoms
d. Intrauterine device (IUD)

A

c. Male or female condoms

Rationale: Only the barrier methods provide some protection from sexually transmitted diseases.
Because latex condoms provide the best protection available, they should be used during any potential exposure to a sexually transmitted disease.
Oral contraceptives, tubal ligations, or IUDs do not provide protection against STDs.

How well did you know this?
1
Not at all
2
3
4
5
Perfectly
36
Q

A couple is discussing alternatives for pregnancy prevention and has asked about fertility awareness methods (FAMs). The nurse’s most appropriate reply is

a. “They’re not very effective, and it’s very likely you’ll get pregnant.”
b. “They can be effective for many couples, but they require motivation.”
c. “These methods have a few advantages and several health risks.”
d. “You would be much safer going on the pill and not having to worry.”

A

b. “They can be effective for many couples, but they require motivation.”

Rationale: FAMs are effective with proper vigilance about ovulatory changes in the body and with adherence to coitus intervals.
However, the typical failure rate is 25%.
This is not the best response, however.
The nurse should provide positive feedback first; otherwise, the couple may become discouraged and think the nurse is negative or biased against a method they are interested in. FAMs have no associated health risks.
The use of birth control has associated health risks.
In addition, taking a pill daily requires compliance on the patient’s part.

How well did you know this?
1
Not at all
2
3
4
5
Perfectly
37
Q

A woman who has a seizure disorder and takes barbiturates and phenytoin sodium daily asks the nurse about the pill as a contraceptive choice. The nurse’s most appropriate response is

a. “This is a highly effective method, but it has some side effects.”
b. “Your current medications will reduce the effectiveness of the pill.”
c. “The pill will reduce the effectiveness of your seizure medication.”
d. “This is a good choice for a woman of your age and personal history.”

A

b. “Your current medications will reduce the effectiveness of the pill.”

Rationale: Because the liver metabolizes oral contraceptives, their effectiveness is reduced when they are taken simultaneously with anticonvulsants.
Telling the woman the pill has some side effects or that it is a good choice for some women is not tailoring teaching to her specific situation.
The anticonvulsant will reduce the effectiveness of the pill, not the other way around

How well did you know this?
1
Not at all
2
3
4
5
Perfectly
38
Q

Injectable progestins (DMPA, Depo-Provera) are a good contraceptive choice for women who

a. want menstrual regularity and predictability.
b. have a history of thrombotic problems or breast cancer.
c. have difficulty remembering to take oral contraceptives daily.
d. are homeless or mobile and rarely receive health care.

A

c. have difficulty remembering to take oral contraceptives daily.

Rationale: Advantages of DMPA include a contraceptive effectiveness comparable to that of combined oral contraceptives with the requirement of only four injections a year.
Disadvantages of injectable progestins are menstrual irregularities.
Use of injectable progestin carries an increased risk of venous thrombosis and thromboembolism.
To be effective, DMPA injections must be administered every 11 to 13 weeks.
Access to health care is necessary to prevent pregnancy or potential complications.

How well did you know this?
1
Not at all
2
3
4
5
Perfectly
39
Q

Which woman is the safest candidate for the use of oral contraceptives?

a. 39-year-old with a history of thrombophlebitis
b. 16-year-old with a benign liver tumor
c. 20-year-old who suspects she may be pregnant
d. 43-year-old who does not smoke cigarettes

A

d. 43-year-old who does not smoke cigarettes

Rationale: Cigarette smoking is a contraindication, especially in women older than 35.
Oral contraceptives are contraindicated with a history of thrombophlebitis, liver tumors, or pregnancy.

How well did you know this?
1
Not at all
2
3
4
5
Perfectly
40
Q

The role of the nurse in family planning is to

a. advise couples on which contraceptive to use.
b. educate couples on the various methods of contraception.
c. decide on the best method of contraception for the couple.
d. refer the couple to a reliable physician.

A

b. educate couples on the various methods of contraception.

Rationale: The nurse’s role is to provide information to the couple so that they can make an informed decision about family planning.
The nurse should not advise the couple or pick the best method for them, nor does he or she need to refer couples for information about contraceptives.

How well did you know this?
1
Not at all
2
3
4
5
Perfectly
41
Q

What does the nurse know about postcoital emergency contraception with Ella or Next Choice?

a. Requires that the first dose be taken within 72 hours of unprotected intercourse
b. Requires that the woman take second and third doses at 24 and 36 hours after the first dose
c. Must be taken in conjunction with an IUD insertion
d. Most states require the woman to have a valid prescription

A

a. Requires that the first dose be taken within 72 hours of unprotected intercourse

Rationale: Emergency contraception is most effective when used within 72 hours of intercourse but may be used with lessened effectiveness up to 120 hours later.
Insertion of the copper IUD within 5 days of intercourse may also be used and is up to 99% effective.
Emergency contraception is available without a prescription for women over 17 and for those younger than 17 with prescription

How well did you know this?
1
Not at all
2
3
4
5
Perfectly
42
Q

Informed consent concerning contraceptive use is important because some of the methods

a. are invasive procedures that require hospitalization.
b. require a surgical procedure to insert.
c. may not be reliable.
d. have potentially dangerous side effects.

A

d. have potentially dangerous side effects.

Rationale: It is important for couples to be aware of potential side effects so they can make an informed decision about the use of contraceptives.
The only contraceptive method that requires hospitalization is sterilization.
The only surgical procedure used would be for permanent sterilization.
Some have more effective rates, and this should be included in the teaching.

How well did you know this?
1
Not at all
2
3
4
5
Perfectly
43
Q

Which contraceptive method is contraindicated in a woman with a history of toxic shock syndrome?

a. Condom
b. Spermicide
c. Cervical cap
d. Norplant

A

c. Cervical cap

Rationale: The cervical cap may increase the risk of toxic shock syndrome because it may be left in the vagina for a prolonged period.
Condoms, spermicides, and Norplant are not contraindicated in women who have had toxic shock syndrome.

How well did you know this?
1
Not at all
2
3
4
5
Perfectly
44
Q

What is important in instructing a patient in the use of spermicidal foams or gels?

a. Insert 1 to 2 hours before intercourse.
b. One application is effective for several hours.
c. Avoid douching for at least 6 hours.
d. There are no known side effects.

A

c. Avoid douching for at least 6 hours.

Rationale: Douching within 6 hours of intercourse removes the spermicide and increases the risk of pregnancy.
Foams or gels should be inserted just before intercourse and are effective for about 1 hour.
Each application is effective for about 1 hour.
Effectiveness is about 74% when used alone.
Vaginal irritation may occur with spermicide use.

How well did you know this?
1
Not at all
2
3
4
5
Perfectly
45
Q

A woman currently uses a diaphragm and spermicide for contraception. She asks the nurse what the major differences are between the cervical cap and diaphragm. The nurse’s most appropriate response is

a. “No spermicide is used with the cervical cap, so it’s less messy.”
b. “The diaphragm can be left in place longer after intercourse.”
c. “Repeated intercourse with the diaphragm is more convenient.”
d. “You can have intercourse several times without removing the cap to add more spermicide.”

A

d. “You can have intercourse several times without removing the cap to add more spermicide.”

Rationale: The cervical cap can be inserted hours before sexual intercourse without the need for additional spermicide later. No additional spermicide is needed inside the cap for repeated acts of intercourse but more is inserted into the vagina. Spermicide should be used inside the cap as an additional chemical barrier.
The cervical cap should remain in place for 6 hours after the last act of intercourse but can stay in place up to 48 hours. Repeated intercourse with the cervical cap is more convenient, because no additional spermicide is needed.

How well did you know this?
1
Not at all
2
3
4
5
Perfectly
46
Q

A woman will be taking oral contraceptives using a 28-day pack. The nurse should advise this woman to protect against pregnancy by

a. limiting sexual contact for one cycle after starting the pill.
b. using condoms and foam instead of the pill for as long as she takes an antibiotic.
c. taking one pill at the same time every day.
d. using a backup method if she misses two pills during week 1 of her cycle.

A

c. taking one pill at the same time every day.

Rationale: To maintain adequate hormone levels for contraception and to enhance compliance, patients should take oral contraceptives at the same time each day.
If contraceptives are to be started at any time other than during normal menses or within 3 weeks after birth or abortion, another method of contraception should be used through the first week to prevent the risk of pregnancy.
Taken exactly as directed, oral contraceptives prevent ovulation, and pregnancy cannot occur.
No strong pharmacokinetic evidence indicates a link between the use of broad-spectrum antibiotics and altered hormonal levels in oral contraceptive users.
If the patient misses two pills during week 1, she should take two pills a day for 2 days and finish the package and use a backup method for the next 7 consecutive days.

How well did you know this?
1
Not at all
2
3
4
5
Perfectly
47
Q

With regard to the use of intrauterine devices (IUDs), nurses should be aware that

a. return to fertility can take several weeks after the device is removed.
b. IUDs containing copper can provide an emergency contraception option if inserted within a few days of unprotected intercourse.
c. IUDs offer the same protection against sexually transmitted diseases as the diaphragm.
d. consent forms are not needed for IUD insertion.

A

b. IUDs containing copper can provide an emergency contraception option if inserted within a few days of unprotected intercourse.

Rationale: The woman has up to 5 days to insert the IUD after unprotected sex. Return to fertility is immediate after removal of the IUD.
IUDs offer no protection for sexually transmitted diseases.
A consent form and a negative pregnancy test are required for insertion.

How well did you know this?
1
Not at all
2
3
4
5
Perfectly
48
Q

The karyotype of a person is 47, XY, +21. This person is a

a. Normal male
b. Male with Down syndrome
c. Normal female
d. Female with Turner syndrome

A

b. Male with Down syndrome

Rationale: This person is male because his sex chromosomes are XY.
He has one extra copy of chromosome 21 (for a total of 47 instead of 46), resulting in Down syndrome.

How well did you know this?
1
Not at all
2
3
4
5
Perfectly
49
Q

People who have two copies of the same abnormal autosomal dominant gene will usually be

a. More severely affected by the disorder than will people with one copy of the gene
b. Infertile and unable to transmit the gene
c. Carriers of the trait but not affected with the disorder
d. Mildly affected with the disorder

A

a. More severely affected by the disorder than will people with one copy of the gene

Rationale: People who have two copies of an abnormal gene are usually more severely affected by the disorder because they have no normal gene to maintain normal function.

How well did you know this?
1
Not at all
2
3
4
5
Perfectly
50
Q

A baby is born with blood type AB. The father is type A, and the mother is type B. The father asks why the baby has a blood type different from those of his parents. The nurses answer should be based on the knowledge that

a. Both A and B blood types are dominant.
b. The baby has a mutation of the parents blood types.
c. Type A is recessive and links more easily with type B.
d. Types A and B are recessive when linked together.

A

a. Both A and B blood types are dominant.

Rationale: Rationale: Types A and B are equally dominant, and the baby can thus inherit one from each parent.

How well did you know this?
1
Not at all
2
3
4
5
Perfectly
51
Q

Which statement is true of multifactorial disorders?

a. They may not be evident until later in life.
b. They are usually present and detectable at birth.
c. The disorders are characterized by multiple defects.
d. Secondary defects are rarely associated with multifactorial disease.

A

b. They are usually present and detectable at birth.

Rationale: Multifactorial disorders result from an interaction between a persons genetic susceptibility and environmental conditions that favor development of the defect.
They are characteristically present and detectable at birth.

How well did you know this?
1
Not at all
2
3
4
5
Perfectly
52
Q

Both members of an expectant couple are carriers for phenylketonuria (PKU), an autosomal recessive disorder. In counseling them about the risk to their unborn child, the nurse should tell them that

a. The child has a 25% chance of being affected.
b. The child will be a carrier, like the parents.
c. The child has a 50% chance of being affected.
d. One of four of their children will be affected.

A

a. The child has a 25% chance of being affected.

Rationale: Each child born to a couple who carries an autosomal recessive trait has a 25% chance of having the disorder, because the child receives either a normal or an abnormal gene from each parent.

How well did you know this?
1
Not at all
2
3
4
5
Perfectly
53
Q

Which statement should a counselor make when telling a couple about the prenatal diagnosis of genetic disorders?

a. Diagnosis can be obtained promptly through most hospital laboratories.
b. Common disorders can quickly be diagnosed through blood tests.
c. A comprehensive evaluation will result in an accurate diagnosis.
d. Diagnosis may be slow and could be inconclusive.

A

d. Diagnosis may be slow and could be inconclusive.

Rationale: Even the best efforts at diagnosis do not always yield the information needed to counsel patients.
They may require many visits over several weeks.

How well did you know this?
1
Not at all
2
3
4
5
Perfectly
54
Q

A woman tells the nurse at a prenatal interview that she has quit smoking, only has a glass of wine with dinner, and has cut down on coffee to four cups a day. What response by the nurse will be most helpful in promoting a lifestyle change?

a. You have made some good progress toward having a healthy baby. Lets talk about the changes you have made.
b. You need to do a lot better than that. You are still hurting your baby.
c. Here are some pamphlets for you to study. They will help you find more ways to improve.
d. Those few things wont cause any trouble. Good for you.

A

a. You have made some good progress toward having a healthy baby. Lets talk about the changes you have made.

Rationale: Praising her for making positive changes is an effective technique for motivating a patient.
She still has to identify the risk factors to optimize the results.

How well did you know this?
1
Not at all
2
3
4
5
Perfectly
55
Q

A 35-year-old woman has an amniocentesis performed to find out whether her baby has a chromosome defect. Which statement by this patient indicates that she understands her situation?

a. The doctor will tell me if I should have an abortion when the test results come back.
b. I know support groups exist for parents who have a baby with birth defects, but we have plenty of insurance to cover what we need.
c. When all the lab results come back, my husband and I will make a decision about the pregnancy.
d. My mother must not find out about all this testing. If she does, she will think Im having an abortion.

A

c. When all the lab results come back, my husband and I will make a decision about the pregnancy.

Rationale: The final decision about genetic testing and the future of the pregnancy lies with the patient.
She will involve only those people whom she chooses in her decisions.

How well did you know this?
1
Not at all
2
3
4
5
Perfectly
56
Q

Which question by the nurse will most likely promote sharing of sensitive information during a genetic counseling interview?

a. How many people in your family are mentally retarded or handicapped?
b. What kinds of defects or diseases seem to run in the family?
c. Did you know that you can always have an abortion if the fetus is abnormal?
d. Are there any family members who have learning or developmental problems?

A

d. Are there any family members who have learning or developmental problems?

Rationale: The nurse should probe gently by using lay-oriented terminology rather than direct questions or statements.

How well did you know this?
1
Not at all
2
3
4
5
Perfectly
57
Q

You are a maternal-newborn nurse caring for a mother who just delivered a baby born with Down syndrome. What nursing diagnosis is the most essential in caring for the mother of this infant?

a. Disturbed body image
b. Interrupted family processes
c. Anxiety
d. Risk for injury

A

b. Interrupted family processes

Rationale: This mother likely will experience a disruption in the family process related to the birth of a baby with an inherited disorder.
Family disruption is common, and the strain of having a child with a serious birth defect may lead to divorce.
Siblings may feel neglected because the child with a disorder requires more of their parents time and attention.

How well did you know this?
1
Not at all
2
3
4
5
Perfectly
58
Q

A couple has been counseled for genetic anomalies. They ask you, What is karyotyping? Your best response is

a. Karyotyping will reveal if the babys lungs are mature.
b. Karyotyping will reveal if your baby will develop normally.
c. Karyotyping will provide information about the gender of the baby and the number and structure of the chromosomes.
d. Karyotyping will detect any physical deformities the baby has.

A

c. Karyotyping will provide information about the gender of the baby and the number and structure of the chromosomes.

Rationale: Karyotyping provides genetic information, such as gender and chromosomal structure.
Karyotyping is completed by photographing or using computer imaging to arrange chromosomes in pairs from largest to smallest.
The karyotype can then be analyzed.

How well did you know this?
1
Not at all
2
3
4
5
Perfectly
59
Q

In practical terms regarding genetic health care, nurses should be aware that

a. Genetic disorders equally affect people of all socioeconomic backgrounds, races, and ethnic groups.
b. Genetic health care is more concerned with populations than individuals.
c. The most important of all nursing functions is providing emotional support to the family during counseling.
d. Taking genetic histories is the province of large universities and medical centers.

A

c. The most important of all nursing functions is providing emotional support to the family during counseling.

Rationale: Nurses should be prepared to help with a variety of stress reactions from a couple facing the possibility of a genetic disorder.

How well did you know this?
1
Not at all
2
3
4
5
Perfectly
60
Q

The nurse is working in an OB/GYN office, where part of her duties include obtaining a patient’s history and performing an initial assessment. Which woman is likely to be referred for genetic counseling after her first visit?

a. A pregnant woman who will be 40 years or older when her infant is born
b. A woman whose partner is 38 years of age
c. A patient who carries a Y-linked disorder
d. An anxious woman with a normal quadruple screening result

A

b. A woman whose partner is 38 years of age

Rationale: Men who father children after the age of 40 should also be sent for referral.
Men who father children in their 5th decade or later are more likely to have offspring with a new autosomal dominant mutation.

How well did you know this?
1
Not at all
2
3
4
5
Perfectly
61
Q

Chromosomes are composed of genes, which are composed of DNA. Abnormalities are either numerical or structural in nature. Which abnormalities are structural? Select all that apply.

a. Part of a chromosome is missing.
b. The material within a chromosome is rearranged.
c. One or more sets of chromosomes are added.
d. Entire single chromosome is added.
e.Two chromosomes adhere to each other.

A

a. Part of a chromosome is missing.
b. The material within a chromosome is rearranged.
e.Two chromosomes adhere to each other.

How well did you know this?
1
Not at all
2
3
4
5
Perfectly
62
Q

A female college student is planning to become sexually active. She is considering birth control options and desires a method in which ovulation will be prevented. To prevent ovulation while reaching 99% effectiveness in preventing pregnancy, which option should be given the strongest consideration?

a. Intrauterine device
b. Coitus interruptus
c. Natural family planning
d. Oral contraceptive pills

A

d. Oral contraceptive pills

Rationale: Oral contraceptive pills prevent ovulation and are 99% effective in preventing pregnancy when taken as directed. Intrauterine devices, coitus interruptus, and natural family planning will not prevent ovulation while reaching 99% effectiveness in preventing pregnancy, so they are not recommended for this college student.

How well did you know this?
1
Not at all
2
3
4
5
Perfectly
63
Q

The nurse at the family planning clinic conducts a male history for infertility evaluation. Which finding has the greatest
implication for this patient’s care?

a. Practice of nightly masturbation
b. Primary anovulation
c. High testosterone levels
d. Impotence due to alcohol ingestion

A

d. Impotence due to alcohol ingestion

Rationale: Factors affecting male infertility include impotence due to alcohol.
Nightly masturbation and high testosterone levels do not have
the greatest implication on male infertility in a patient with admitted alcohol issues.
Primary anovulation refers to female infertility, so it is not a consideration for male infertility.

How well did you know this?
1
Not at all
2
3
4
5
Perfectly
64
Q

The emergency department nursing assessment of a pregnant female at 35 weeks gestation reveals back pain, blood pressure 150/92, and leaking of clear fluid from the vagina. Which complication of pregnancy does the nurse suspect?

a. Ectopic pregnancy
b. Spontaneous abortion
c. Premature rupture of membranes
d. Supine hypotension

A

c. Premature rupture of membranes

Rationale: Leaking of clear fluid from the vagina with back pain and elevated BP is associated with premature rupture of membranes, a complication of pregnancy.
An ectopic pregnancy usually manifests as unilateral pain early in the pregnancy.
Vaginal bleeding is a classic sign of miscarriage, or spontaneous abortion, not leaking of clear fluid.
This patient’s blood pressure is elevated.
Supine hypotension occurs when the woman is lying supine; then low blood pressure occurs due to the decrease in venous return from the gravid uterus placing pressure on the vena cava.

How well did you know this?
1
Not at all
2
3
4
5
Perfectly
65
Q

The nurse is admitting a prenatal patient for diagnostic testing. While eliciting the psychosocial history, the nurse learns the patient smokes a pack of cigarettes daily, drinks a cup of cappuccino with breakfast, has smoked marijuana in the remote past, and is a social drinker. Which action should the nurse first take?

a. Strongly advise immediate tobacco cessation
b. Elimination of all caffeinated beverages
c. Serum and urine testing for drug use and alcohol use
d. Referral to a 12-step program

A

a. Strongly advise immediate tobacco cessation

Rationale: There are numerous risk factors for women and men affecting reproductive health and pregnancy outcomes. These can be categorized into biophysical, psychosocial, sociodemographic, and environmental factors.
Some of the risk factors for human reproduction fit into multiple categories.
Psychosocial factors cover smoking, excessive caffeine, alcohol and drug abuse, psychological status including impaired mental health, addictive lifestyles, spouse abuse, and noncompliance with cultural norms.
Drinking a cup of a caffeinated beverage a day is not associated with adverse fetal outcomes usually.
Serum and urine testing for drug/alcohol use is not required for stated marijuana use in the remote past.
Patient referral to a 12-step program is usually advisable for current alcohol and/or drug use.

How well did you know this?
1
Not at all
2
3
4
5
Perfectly
66
Q

A female infertility patient is found to be hypoestrogenic at the preconceptual clinic visit. She asks the nurse why she has never been able to get pregnant. Which is the best nursing response?

a. Circulating estrogen contributes to secondary sex characteristics.
b. Estrogen deficiency prevents the ovum from reaching the uterus and may be a factor in infertility.
c. Hyperestrogen may be preventing the zona pellucida from forming an ovum protective layer.
d. The corona radiata is preventing fertilization of the ovum.

A

b. Estrogen deficiency prevents the ovum from reaching the uterus and may be a factor in infertility.

Rationale: The cilia in the tubes are stimulated by high estrogen levels, which propel the ovum toward the uterus. Without estrogen, the ovum won’t reach the uterus.
The results of a series of events occurring in the ovary cause an expulsion of the oocyte from the ovarian follicle known as ovulation.
The ovarian cycle is driven by multiple important hormones: (1) gonadotropic hormone, (2) follicle-stimulating hormone (FSH), and (3) luteinizing hormone (LH).
The zona pellucida (inner layer) and corona radiata (outer layer) form protective layers around the ovum.
If an ovum is not fertilized within 24 hours of ovulation by a sperm, it is usually reabsorbed into a woman’s body.
A patient who is hypoestrogenic would not have excess circulating estrogen.
A patient with low estrogen would not be classified as hyperestrogenic.
Without sufficient estrogen, there can be no fertilization of the ovum.

How well did you know this?
1
Not at all
2
3
4
5
Perfectly
67
Q

An obstetric multipara with triplets is placed on bed rest at 24 weeks’ gestation. Her perinatologist is managing intrauterine
growth restriction with serial ultrasounds. This prescribed treatment is an example of which type of care?

a. Antenatal diagnostics
b. Primary prevention
c. Secondary prevention
d. Tertiary prevention

A

c. Secondary prevention

Rationale: An example of secondary prevention relating to reproductive health would be managing fetal intrauterine growth restriction by serial ultrasounds.
This type of diagnostic maternal/fetal monitoring is performed to determine the best time for delivery due to
potential fetal nutritional, circulatory, or pulmonary compromise.
A cesarean section (operative delivery) may be performed if
maternal or fetal conditions indicate that delivery is necessary. Antenatal diagnostics refers to prior to pregnancy.
An example of primary prevention is teaching a high school class about reproductive health.
An example of tertiary prevention would be aimed at improving health following an illness and/or rehabilitation.

How well did you know this?
1
Not at all
2
3
4
5
Perfectly
68
Q

A female patient comes to the clinic after missing one menstrual period. She lives in a house beneath electrical power lines which is located near an oil field. She drinks two caffeinated beverages a day, is a daily beer drinker, and has not stopped eating sweets. She takes a multivitamin and exercises daily. She denies drug use. Which finding in the history has the greatest implication for this patient’s plan of care?

a. Electrical power lines are a potential hazard to the woman and her fetus.
b. Living near an oil field may mean the water supply is polluted.
c. Alcohol exposure should be avoided during pregnancy due to teratogenicity.
d. Eating sweets may cause gestational diabetes or miscarriage.

A

c. Alcohol exposure should be avoided during pregnancy due to teratogenicity.

Rationale: Stages of development include ovum, embryonic, and fetal.
The beginning of the fourth week to the end of the eighth week
comprise the embryonic period.
Teratogenicity is a major concern because all external and internal structures are developing in the embryonic period.
A pregnant woman should avoid exposure to all potential toxins during pregnancy, especially alcohol, tobacco, radiation, and infections during embryonic development.
Living in a house beneath power lines is not the greatest
implication in this patient’s plan of care as there are no definite risks to the developing fetus.
Living near an oil field has no definite risks to the fetus.
Eating sweets may contribute to maternal obesity, large for gestational age fetus, and maternal gestational diabetes but does not have the immediate implication of a daily beer drinker which can cause fetal alcohol syndrome.

How well did you know this?
1
Not at all
2
3
4
5
Perfectly
69
Q

Which man is most likely to have abnormal sperm formation resulting in infertility?

a. A 20-year-old man with undescended testicles
b. An uncircumcised 40-year-old man
c. A 35-year-old man with previously treated sexually transmitted disease
d. A 16-year-old adolescent who is experiencing nocturnal emissions

A

a. A 20-year-old man with undescended testicles

Rationale: For normal sperm formation, a man’s testes must be cooler than his core body temperature.
The cremaster muscle attached to each testicle causes the testes to rise closer to the body and become warmer or allow the testes to fall away from the body to become cooler. Circumcision does not prevent fertility.
Scar tissue in the fallopian tubes as a result of a sexually transmitted disease can be a cause of infertility in women. Nocturnal emissions of seminal fluid are normal and expected in teenagers.

How well did you know this?
1
Not at all
2
3
4
5
Perfectly
70
Q

A nurse is teaching a woman about spinnbarkeit. The student nurse asks why the woman would need this information. What response by the nurse is most appropriate?

a. To assist in becoming pregnant or preventing pregnancy
b. To determine if she can breastfeed
c. To assess risk for genetic defects in the fetus
d. To find out if her ova are suitable for fertilization

A

a. To assist in becoming pregnant or preventing pregnancy

Rationale: Spinnbarkeit refers to the elasticity of cervical mucosa.
The woman can assess this to avoid or promote pregnancy.
It does not refer to breastfeeding, genetics, or her ova status.

How well did you know this?
1
Not at all
2
3
4
5
Perfectly
71
Q

Which of these is a secondary sexual characteristic?

a. Female breast development
b. Production of sperm
c. Maturation of ova
d. Secretion of gonadotropin-releasing hormone

A

a. Female breast development

Rationale: A secondary sexual characteristic is one not directly related to reproduction, such as development of the characteristic female body form.
Production of sperm, maturation of ova, and secretion of hormones are all directly related to reproduction and not secondary sexual characteristics.

How well did you know this?
1
Not at all
2
3
4
5
Perfectly
72
Q

The nursing students learn that fertilization of the ovum takes place in which part of the fallopian tube?

a. Interstitial portion
b. Ampulla
c. Isthmus
d. Infundibulum

A

b. Ampulla

Rationale: The ampulla is the wider middle part of the tube lateral to the isthmus and is where fertilization occurs.
It does not occur in the interstitial portion, isthmus, or infundibulum.

How well did you know this?
1
Not at all
2
3
4
5
Perfectly
73
Q

Which 16-year-old female is most likely to experience secondary amenorrhea?

a. A girl who is 5 ft 2 in, 130 lb
b. A girl who is 5 ft 9 in, 150 lb
c. A girl who is 5 ft 7 in, 96 lb
d. A girl who is 5 ft 4 in, 120 lb

A

c. A girl who is 5 ft 7 in, 96 lb

Rationale: Low body fat is a risk factor for secondary amenorrhea.
The girl who is 5 ft 7 inches tall and only weighs 96 pounds has less body fat that the other girls and a higher likelihood of secondary amenorrhea.

How well did you know this?
1
Not at all
2
3
4
5
Perfectly
74
Q

It is important for the nurse to understand that the levator ani is a(n)

a. imaginary line that divides the true and false pelvis.
b. basin-shaped structure at the lower end of the spine.
c. collection of three pairs of muscles.
d. division of the fallopian tube.

A

c. collection of three pairs of muscles.

Rationale: The levator ani is a collection of three pairs of muscles that support internal pelvic structures and resist increases in intra-abdominal pressure.
The linea terminalis is the imaginary line that divides the false pelvis from the true pelvis.
The basin-shaped structure at the lower end of the spine is the bony pelvis.
The fallopian tube divisions are the interstitial portion, isthmus, ampulla, and infundibulum.

How well did you know this?
1
Not at all
2
3
4
5
Perfectly
75
Q

In describing the size and shape of the nonpregnant uterus to a patient, the nurse would say it is approximately the size and shape of a

a. cantaloupe.
b. grapefruit.
c. pear.
d. large orange.

A

c. pear.

Rationale: The nonpregnant uterus is approximately 7.5 × 5.0 × 2.5 cm, which is close to the size and shape of a pear.

How well did you know this?
1
Not at all
2
3
4
5
Perfectly
76
Q

If a woman’s menstrual cycle began on June 2 and normally lasts 28 days, ovulation would mostly likely occur on June

a. 10
b. 16
c. 21
d. 29

A

b. 16

Rationale: Ovulation occurs approximately 12 to 14 days after the beginning of the menstrual period in a 28-day cycle.
In this woman, ovulation would most likely occur on June 16. June 10 would just be 8 days into the cycle and too early for ovulation.
June 21 would be 18 days into the cycle.
Ovulation should have already occurred at this point.
June 29 would be 27 days into the cycle and almost time for the next period.

How well did you know this?
1
Not at all
2
3
4
5
Perfectly
77
Q

A patient states, “My breasts are so small, I don’t think I will be able to breastfeed.” The nurse’s best response is

a. “It may be difficult, but you should try anyway.”
b. “You can always supplement with formula.”
c. “Breast size is not related to the ability to breastfeed.”
d. “The ability to breastfeed depends on secretion of estrogen and progesterone.”

A

c. “Breast size is not related to the ability to breastfeed.”

Rationale: All women have approximately the same amount of glandular tissue to secrete milk, despite breast size.
Saying that nursing will be difficult or that the woman can use formula does not provide the woman with accurate information.
Increased estrogen decreases the production of milk.

How well did you know this?
1
Not at all
2
3
4
5
Perfectly
78
Q

The function of the cremaster muscle in men is to

a. aid in voluntary control of excretion of urine.
b. entrap blood in the penis to produce an erection.
c. assist with transporting sperm.
d. aid in temperature control of the testicles.

A

d. aid in temperature control of the testicles.

Rationale: A cremaster muscle is attached to each testicle.
Its function is to bring the testicle closer to the body to warm it or allow it to fall away from the body to cool it, thus promoting normal sperm production.
It is not involved in urination, causing an erection or assist in transporting sperm.

How well did you know this?
1
Not at all
2
3
4
5
Perfectly
79
Q

The average man is taller than the average woman at maturity because of

a. a longer period of skeletal growth.
b. earlier development of secondary sexual characteristics.
c. earlier onset of growth spurt.
d. starting puberty at an earlier age.

A

a. a longer period of skeletal growth.

Rationale: The man’s greater height at maturity is the combined result of beginning the growth spurt at a later age and continuing it for a longer period.
Girls develop earlier than boys.
Boys’ growth spurts start at a later age.
Girls start puberty approximately 6 months to 1 year earlier than boys.

How well did you know this?
1
Not at all
2
3
4
5
Perfectly
80
Q

A student nurse just read that up to 200 million sperm are deposited in the vagina with each ejaculation and asks the faculty why so many are needed. What response by the faculty is most accurate?

a. Competition results in fewer genetic defects.
b. Sperm are weak and die off quickly.
c. Few sperm reach the fallopian tube and ova.
d. Most sperm are not the correct shape.

A

c. Few sperm reach the fallopian tube and ova.

Rationale: Although a huge quantity of sperm are released with each ejaculation, very few make it to the fallopian tube where an ovum may be waiting to be fertilized.

How well did you know this?
1
Not at all
2
3
4
5
Perfectly
81
Q

The student nurse learns that follicle-stimulating hormone is produced in which gland?

a. Anterior pituitary
b. Posterior pituitary
c. Hypothalamus
d. Adrenal glands

A

a. Anterior pituitary

Rationale: Follicle stimulating hormone is produced in the anterior pituitary gland.

How well did you know this?
1
Not at all
2
3
4
5
Perfectly
82
Q

A young female patient comes to the school nurse to discuss her irregular periods. In providing education regarding the female reproductive cycle, which phases of the ovarian cycle does the nurse include? (Select all that apply.)

a. Follicular
b. Ovulatory
c. Luteal
d. Proliferative
e. Secretory

A

a. Follicular
b. Ovulatory
c. Luteal

Rationale: The follicular phase is the period during which the ovum matures. It begins on day 1 and ends around day 14.
The ovulatory phase occurs near the middle of the cycle, approximately 2 days before ovulation.
After ovulation and under the influence of the luteinizing hormone, the luteal phase corresponds with the last 12 days of the menstrual cycle.
The proliferative and secretory phases are part of the endometrial cycle.
The proliferative phase takes place during the first half of the ovarian cycle when the ovum matures.
The secretory phase occurs during the second half of the cycle when the uterus is prepared to accept the fertilized ovum. These are followed by the menstrual phase if fertilization does not occur.

83
Q

The anterior pituitary gland is responsible for producing which hormones? (Select all that apply.)

a. Follicle-stimulating hormone (FSH)
b. Luteinizing hormone (LH)
c. Gonadotropin-releasing hormone (GnRH)
d. Oxytocin
e. Prolactin

A

a. Follicle-stimulating hormone (FSH)
b. Luteinizing hormone (LH)
e. Prolactin

Rationale: FSH and LH are both produced by the anterior pituitary gland.
Both of these hormones assist in the stimulation and maturation of the ovarian follicle.
Prolactin is also produced by the anterior pituitary and is required for milk production (lactogenesis) to occur.
GnRH is produced by the hypothalamus and stimulates the release of FSH and LH.
Oxytocin is produced by the posterior pituitary gland and is responsible for stimulating uterine contractions during birth

84
Q

Which part of the mature sperm contains the male chromosomes?

a. The head of the sperm
b. The middle portion of the sperm
c. X-bearing sperm
d. The tail of the sperm

A

a. The head of the sperm

Rationale: The head of the sperm contains the male chromosomes that will join the chromosomes of the ovum. The middle portion of the sperm supplies energy for the tail’s whip-like action.
If an X-bearing sperm fertilizes the ovum, the baby will be female.
The tail of the sperm helps propel the sperm toward the ovum.

85
Q

One of the assessments performed in the delivery room is checking the umbilical cord for blood vessels. Which finding is considered within normal limits?

a. Two arteries and one vein
b. Two arteries and two veins
c. Two veins and one artery
d. One artery and one vein

A

a. Two arteries and one vein

Rationale: The umbilical cord contains two arteries and one vein to transport blood between the fetus and the placenta. Any option other than two arteries and one vein is considered abnormal and requires further assessment.

86
Q

The purpose of the ovum’s zona pellucida is to

a. make a pathway for more than one sperm to reach the ovum.
b. allow the 46 chromosomes from each gamete to merge.
c. prevent multiple sperm from fertilizing the ovum.
d. stimulate the ovum to begin mitotic cell division.

A

c. prevent multiple sperm from fertilizing the ovum.

Rationale: Fertilization causes the zona pellucida to change its chemical composition so that multiple sperm cannot fertilize the ovum.
Each gamete (sperm and ovum) has only 23 chromosomes. There will be 46 chromosomes when they merge.
Mitotic cell division begins when the nuclei of the sperm and ovum unite.

87
Q

The nursing faculty explains to students that the upper uterus is the best place for the fertilized ovum to implant because it is here that the

a. placenta attaches most firmly.
b. developing baby is best nourished.
c. uterine endometrium is softer.
d. maternal blood flow is lower.

A

b. developing baby is best nourished.

Rationale: The uterine fundus is richly supplied with blood and has the thickest endometrium, both of which promote optimal nourishment of the fetus.
If the placenta attaches too deeply, it does not easily detach after birth.
Softness is not a concern with implantation; attachment and nourishment are the major concerns.
The blood supply is rich in the fundus, which allows for optimal nourishment of the fetus.

88
Q

The student nurse learns that some of the embryo’s intestines remain within the umbilical cord during the embryonic period because the

a. umbilical cord is much larger at this time than it will be at the end of pregnancy.
b. intestines begin their development within the umbilical cord.
c. nutrient content of the blood is higher in this location.
d. abdomen is too small to contain all the organs while they are developing.

A

d. abdomen is too small to contain all the organs while they are developing.

Rationale: The abdominal contents grow more rapidly than the abdominal cavity, so part of their development takes place in the umbilical cord.
By 10 weeks, the abdomen is large enough to contain them. The intestines remain within the umbilical cord only until approximately week 10.
Intestines begin their development within the umbilical cord but only because the liver and kidneys occupy most of the abdominal cavity.
All the intestines are within the abdominal cavity around week 10.

89
Q

A woman is 16 weeks pregnant with her first baby. She asks how long it will be before she feels the baby move. The best answer is

a. “You should have felt the baby move by now.”
b. “Within the next month, you should start to feel fluttering sensations.”
c. “The baby is moving, but you can’t feel it yet.”
d. “Some babies are quiet, and you don’t feel them move.”

A

b. “Within the next month, you should start to feel fluttering sensations.”

Rationale: Maternal perception of fetal movement usually begins 17 to 20 weeks after conception, particularly if this is the first pregnancy.
“The baby is moving, but you can’t feel it yet” is a true statement.
The fetus’s movements are not strong enough to be felt until 17 to 20 weeks; however, this statement does not answer the concern of the woman.
If no movement is felt at the end of 20 weeks, further assessment is needed.

90
Q

During a pregnancy group meeting, the nurse teaches patients that the fetal period is best described as one of

a. development of basic organ systems.
b. resistance of organs to damage from external agents.
c. maturation of organ systems.
d. development of placental oxygen-carbon dioxide exchange.

A

c. maturation of organ systems.

Rationale: During the fetal period, the body systems grow in size and mature in function to allow independent existence after birth.
Basic organ systems are developed during the embryonic period.
The organs are always at risk for damage from external sources; however, the older the fetus, the more resistant the organs will be.
The greatest risk is when the organs are developing.
The placental system is complete by week 12, but that is not the best description of the fetal period.

91
Q

A new mother is distresses over the “white substance” covering her infant because it “looks ugly.” What action by the nurse is most appropriate?

a. Scrub the substance off of the baby.
b. Reassure the mom that it will go away.
c. Report the findings to the provider.
d. Explain that the vernix caseosa protects fetal skin from amniotic fluid.

A

d. Explain that the vernix caseosa protects fetal skin from amniotic fluid.

Rationale: Prolonged exposure to amniotic fluid during the fetal period could result in breakdown of the skin without the protection of the vernix caseosa.
This can be washed off gently, when the baby gets the first bath.
Although it will not remain, this statement does not explain the purpose of the substance.
This does not need to be reported.

92
Q

A woman who is 16 weeks pregnant asks the nurse, “Is it possible to tell by ultrasound if the baby is a boy or girl yet?” The best answer is

a. “A baby’s sex is determined as soon as conception occurs, and the differences are apparent.”
b. “The baby has developed enough that we can determine the sex by examining the genitals through ultrasound.”
c. “Boys and girls look alike until approximately 20 weeks after conception, and then they begin to look different.”
d. “It might be possible to determine your baby’s sex, but the external organs look very similar right now.”

A

b. “The baby has developed enough that we can determine the sex by examining the genitals through ultrasound.”

Rationale: Although gender is determined at conception, the external genitalia of males and females look similar through the 9th week.
By the 12th week, the external genitalia are distinguishable as male or female.

93
Q

The placenta allows exchange of oxygen, nutrients, and waste products between the mother and fetus by

a. contact between maternal blood and fetal capillaries within the chorionic villi.
b. interaction of maternal and fetal pH levels within the endometrial vessels.
c. a mixture of maternal and fetal blood within the intervillous spaces.
d. passive diffusion of maternal carbon dioxide and oxygen into the fetal capillaries.

A

a. contact between maternal blood and fetal capillaries within the chorionic villi.

Rationale: Fetal capillaries within the chorionic villi are bathed with oxygen- and nutrient-rich maternal blood within the intervillous spaces.
The endometrial vessels are part of the uterus.
There is no interaction with the fetal blood at this point. Maternal and fetal blood do not normally mix.
Maternal carbon dioxide does not enter into the fetal circulation.

94
Q

A patient is sent from the physician’s office for assessment of oligohydramnios. The nurse is aware that this condition can result in

a. excessive fetal urine secretion.
b. newborn respiratory distress.
c. central nervous system abnormality.
d. gastrointestinal blockage.

A

b. newborn respiratory distress.

Rationale: Because an abnormally small amount of amniotic fluid restricts normal lung development, the infant may have inadequate respiratory function after birth, when the placenta no longer performs respiratory function.
Oligohydramnios may be caused by a decrease in urine secretion.
Excessive amniotic fluid production may occur when the fetus has a central nervous system abnormality.
Excessive amniotic fluid production may occur when the gastrointestinal tract prevents normal ingestion of amniotic fluid.

95
Q

When explaining twin conception, the nurse points out that dizygotic twins develop from

a. a single fertilized ovum and are always of the same sex.
b. a single fertilized ovum and may be the same sex or different sexes.
c. two fertilized ova and are the same sex.
d. two fertilized ova and may be the same sex or different sexes.

A

d. two fertilized ova and may be the same sex or different sexes.

Rationale: Dizygotic twins are two different zygotes, each conceived from a single ovum and a single sperm.
They may be both male, both female, or one male and one female.
A single fertilized ovum that produces twins is called monozygotic.
Monozygotic twins are always the same sex.
Dizygotic twins are from two fertilized ova and may or may not be the same sex.

96
Q

Which statement related to oogenesis is correct?

a. Two million primary oocytes will mature.
b. At birth, all ova are contained in the female’s ovaries.
c. The oocytes complete their division during fetal life.
d. Monthly, at least two oocytes mature.

A

b. At birth, all ova are contained in the female’s ovaries.

Rationale: All of the cells that may undergo meiosis in a woman’s lifetime are contained in the ovaries at birth.
Only 400 to 500 ova will mature during the approximately 35 years of a woman’s reproductive life.
The primary oocytes begin their first meiotic division during fetal life but remain suspended until puberty.
Every month, one primary oocyte matures and completes meiotic division yielding two unequal cells.

97
Q

After implantation, tiny projections develop out of the trophoblast and extend into the endometrium. These projections are referred to as

a. decidua basalis.
b. decidua capsularis.
c. decidua vera.
d. chorionic villi.

A

d. chorionic villi.

Rationale: These villi are vascular processes that obtain oxygen and nutrients from the maternal bloodstream and dispose of carbon dioxide and waste products into the maternal blood. The deciduas basalis is the portion of the endometrium where the chorionic villi tap into the maternal blood vessels.
The deciduas capsularis is the portion of the endometrium that covers the blastocyst.
The portion of the endometrium that lines the rest of the uterus is called decidua vera.

98
Q

A nurse is teaching a prenatal class. The nurse teaches that during weeks 25 to 28, which fetal development occurs?

a. Eyes reopen.
b. Vernix caseosa covers the skin.
c. Lanugo may develop.
d. Brown fat is deposited.

A

a. Eyes reopen.

Rationale: During this time frame the eyes reopen, and the fetus becomes plumper with smoother skin.
The other changes occur during weeks 17 to 20.

99
Q

A young patient comes in for her first prenatal examination. This is her first child. She asks “How does my baby get air inside my uterus?” The correct response is

a. “The baby’s lungs work in the uterus to exchange oxygen and carbon dioxide.”
b. “The baby absorbs oxygen from your blood system.”
c. “The placenta provides oxygen to the baby and excretes carbon dioxide into your bloodstream.”
d. “The placenta delivers oxygen-rich blood through the umbilical artery to the baby’s abdomen.”

A

c. “The placenta provides oxygen to the baby and excretes carbon dioxide into your bloodstream.”

Rationale: The placenta delivers oxygen-rich blood through the umbilical vein, not the artery, to the fetus and excretes carbon dioxide into the maternal bloodstream.
The fetal lungs do not function as respiratory gas exchange in utero.
The baby does not simply absorb oxygen from a woman’s blood system; rather, blood and gas transport occur through the placenta.

100
Q

The most basic information a maternity nurse should have concerning conception is

a. ova are considered fertile 48 to 72 hours after ovulation.
b. sperm remain viable in the woman’s reproductive system for an average of 12 to 24 hours.
c. conception is achieved when a sperm successfully penetrates the membrane surrounding the ovum.
d. implantation in the endometrium occurs 6 to 10 days after conception.

A

d. implantation in the endometrium occurs 6 to 10 days after conception.

Rationale: Implantation occurs 6 to 10 days after conception and is complete after 10 days.
Ova are considered fertile for approximately 24 hours after ovulation.
Sperm remain viable in the woman’s reproductive system for an average of 2 to 3 days.
Penetration of the ovum by the sperm is called fertilization. Conception occurs when the zygote, the first cell of the new individual, is formed.

101
Q

With regard to the structure and function of the placenta, the maternity nurse should be aware that

a. as the placenta widens, it gradually thins to allow easier passage of air and nutrients.
b. as one of its early functions, the placenta acts as an endocrine gland.
c. the placenta is able to keep out most potentially toxic substances, such as cigarette smoke, to which the mother is exposed.
d. optimal blood circulation is achieved through the placenta when the woman is lying on her back or standing.

A

b. as one of its early functions, the placenta acts as an endocrine gland.

Rationale: The placenta produces four hormones necessary to maintain the pregnancy.
The placenta widens until week 20 and continues to grow thicker.
Toxic substances such as nicotine and carbon monoxide readily cross the placenta into the fetus.
Optimal circulation occurs when the woman is lying on her side.

102
Q

Which statement is accurate about the development of fetal organs and systems?

a. The cardiovascular system is the first organ system to function in the developing human.
b. Hematopoiesis originating in the yolk sac begins in the liver at 10 weeks.
c. The body changes from straight to C-shaped at 8 weeks.
d. The gastrointestinal system is mature at 32 weeks.

A

a. The cardiovascular system is the first organ system to function in the developing human.

Rationale: The heart is developmentally complete by the end of the embryonic stage.
Hematopoiesis begins in the liver during the 6th week.
The body becomes C-shaped at 21 weeks.
The gastrointestinal system is complete at 36 weeks.

103
Q

What does the student learn about recent trends in multiple births?

a. The rate of twin births has declined.
b. The rate of higher order pregnancies has increased.
c. Higher order pregnancies are now very rare.
d. Twinning is the most common form of multiple pregnancy.

A

d. Twinning is the most common form of multiple pregnancy.

Rationale: Twinning is the most common form of multiple pregnancy, and the rate has been increasing, not declining. Higher-order births increased for a time but have now decreased, although they are not rare.

104
Q

The nurse assesses pregnant women for exposure to human teratogens, including which of the following? (Select all that apply.)

a. Infections
b. Radiation
c. Maternal conditions
d. Drugs
e. Chemicals

A

a. Infections
b. Radiation
c. Maternal conditions
d. Drugs

Rationale: Exposure to radiation and a number of infections may result in profound congenital deformities.
These include varicella, rubella, syphilis, parvovirus, CMV, and toxoplasmosis.
Certain maternal conditions such as diabetes and PKU may also affect organs and other parts of the embryo during this developmental period.
Drugs such as antiseizure medication and some antibiotics, as well as chemicals including lead, mercury, tobacco, and alcohol, also may result in structural and functional abnormalities.
Coffee is not considered a teratogen.

105
Q

The nursing faculty teaches that the placenta produces many hormones necessary for normal pregnancy. These include (Select all that apply.)

a. human chorionic gonadotropin (hCG).
b. insulin.
c. estrogen.
d. progesterone.
e. testosterone.

A

a. human chorionic gonadotropin (hCG).
c. estrogen.
d. progesterone.

Rationale: The placenta produces hCG, estrogen, and progesterone.
It does not produce insulin or testosterone.

106
Q

The student learns about shunts that support fetal circulation. Which of the following are included in this support system? (Select all that apply.)

a. Ductus venosus
b. Foramen ovale
c. Ductus arteriosus
d. Foramen magnum
e. Ductus deferens

A

a. Ductus venosus
b. Foramen ovale
c. Ductus arteriosus

Rationale: The ductus venosus, foramen ovale, and ductus arteriosus are part of fetal circulation.
The foramen magnum is located at the base of the skull.
The ductus (or vas) deferens is part of the male reproductive system.

107
Q

The nursing faculty explains that the fetus can survive in a low-oxygen environment due to which of the following? (Select all that apply.)

a. Fetal hemoglobin carries more oxygen than an adult’s.
b. The fetus has higher average hemoglobin and hematocrit.
c. Hemoglobin carries more oxygen at low partial pressures of carbon dioxide.
d. Fetal blood is more acidic than the maternal blood.
e. The fetus does not need gas exchange while in utero.

A

a. Fetal hemoglobin carries more oxygen than an adult’s.
b. The fetus has higher average hemoglobin and hematocrit.
c. Hemoglobin carries more oxygen at low partial pressures of carbon dioxide.

Rationale: The fetus can survive in low oxygen environments due to its hemoglobin being able to carry more oxygen that the mom, having a higher level of hemoglobin and hematocrit, and the fact that hemoglobin can carry more oxygen at low partial pressures of carbon dioxide.
Fetal blood is alkaline.
The fetus does need gas exchange in utero.

108
Q

A pregnant woman’s mother is worried that her daughter is not “big enough” at 20 weeks. The nurse palpates and measures the fundal height at 20 cm, which is even with the woman’s umbilicus. What should the nurse report to the woman and her mother?

a. “The body of the uterus is at the belly button level, just where it should be at this time.”
b. “You’re right. We’ll inform the practitioner immediately.”
c. “When you come for next month’s appointment, we’ll check you again to make sure that the baby is growing.”
d. “Lightening has occurred, so the fundal height is lower than expected.”

A

a. “The body of the uterus is at the belly button level, just where it should be at this time.”

Rationale: At 20 weeks, the fundus is usually located at the umbilical level.
Because the uterus grows in a predictable pattern, obstetric nurses should know that the uterus of 20 weeks of gestation is located at the level of the umbilicus.
There is no need to inform the practitioner.
The nurse should reassure both mother and patient that the findings are normal.
The descent of the fetal head (lightening) occurs in late pregnancy.

109
Q

While the nurse assesses the vital signs of a pregnant woman in her third trimester, the patient complains of feeling faint, dizzy, and agitated. Which nursing intervention is appropriate?

a. Have the patient stand up and retake her blood pressure.
b. Have the patient sit down and hold her arm in a dependent position.
c. Have the patient lie supine for 5 minutes and recheck her blood pressure on both arms.
d. Have the patient turn to her left side and recheck her blood pressure in 5 minutes.

A

d. Have the patient turn to her left side and recheck her blood pressure in 5 minutes.

Rationale: Blood pressure is affected by positions during pregnancy.
The supine position may cause occlusion of the vena cava and descending aorta.
Turning the pregnant woman to a lateral recumbent position alleviates pressure on the blood vessels and quickly corrects supine hypotension.
Pressures are significantly higher when the patient is standing. This option causes an increase in systolic and diastolic pressures.
The arm should be supported at the same level of the heart. The supine position may cause occlusion of the vena cava and descending aorta, creating hypotension

110
Q

A pregnant woman has come to the emergency department with complaints of nasal congestion and epistaxis. What action by the nurse is best?

a. Refer the patient to an ear, nose, and throat specialist.
b. Explain that nasal stuffiness and nosebleeds are caused by a decrease in progesterone.
c. Attach the woman to a cardiac monitor, and draw blood for hemoglobin and hematocrit.
d. Teach that the increased blood supply to the mucous membranes and can result in congestion and nosebleeds.

A

d. Teach that the increased blood supply to the mucous membranes and can result in congestion and nosebleeds.

Rationale: As capillaries become engorged, the upper respiratory tract is affected by the subsequent edema and hyperemia, which causes these conditions, seen commonly during pregnancy.
No referral is needed.
The patient does not need to be attached to a cardiac monitor or have lab drawn.
The patient should be taught that estrogen causes these changes, not progesterone.

111
Q

Which finding in the urine analysis of a pregnant woman is considered a variation of normal?

a. Proteinuria
b. Glycosuria
c. Bacteria
d. Ketonuria

A

b. Glycosuria

Rationale: Small amounts of glucose may indicate “physiologic spilling,” which occurs because the filtered load exceeds the renal tubules’ ability to absorb them.
The presence of protein could indicate kidney disease or preeclampsia.
Urinary tract infections are associated with bacteria in the urine.
An increase in ketones indicates that the patient is exercising too strenuously or has an inadequate fluid and food intake.

112
Q

Which suggestion is appropriate for the pregnant woman who is experiencing nausea and vomiting?

a. Eat only three meals a day so the stomach is empty between meals.
b. Drink plenty of fluids with each meal.
c. Eat dry crackers or toast before arising in the morning.
d. Drink coffee or orange juice immediately on arising in the morning.

A

c. Eat dry crackers or toast before arising in the morning.

Rationale: This will assist with the symptoms of morning sickness.
It is also important for the woman to arise slowly.
Instruct the woman to eat five to six small meals rather than three full meals per day.
Nausea is more intense when the stomach is empty.
Fluids should be taken separately from meals.
Fluids overstretch the stomach and may precipitate vomiting. Coffee and orange juice stimulate acid formation in the stomach.
It is best to suggest eating dry carbohydrates when rising in the morning.

113
Q

Which statement related to changes in the breasts during pregnancy is the most accurate?

a. During the early weeks of pregnancy there is decreased sensitivity.
b. Nipples and areolae become more pigmented.
c. Montgomery tubercles are no longer visible around the nipples.
d. Venous congestion of the breasts is more visible in the multiparous woman.

A

b. Nipples and areolae become more pigmented.

Rationale: Nipples and areolae become more pigmented, and the nipples become more erectile and may express colostrum. Fullness, heightened sensitivity, tingling, and heaviness of the breasts occur in the early weeks of gestation in response to increased levels of estrogen and progesterone.
Montgomery tubercles may be seen around the nipples.
These sebaceous glands may have a protective role in that they keep the nipples lubricated for breastfeeding.
Venous congestion in the breasts is more obvious in primigravidas.

114
Q

A student nurse reads a patient’s chart and sees the term “striae gravidarum,” The student asks the registered nurse what this means. What response by the nurse is accurate?

a. Stretch marks on the abdomen and breasts
b. Dark pigmentation on the woman’s face
c. Bluish-purple discoloration of the vagina and labia
d. Reddened bleeding gums in a pregnant woman

A

a. Stretch marks on the abdomen and breasts

Rationale: Stretch marks occurring on the abdomen and/or breasts of a pregnant woman are called striae gravidarum. Dark pigmentation on the face is known as melasma, chloasma, or the mask of pregnancy.
The bluish tint to the vagina and labia is known as Chadwick’s sign.
Reddened and bleeding gums are known as gingivitis in both pregnant and non-pregnant women.

115
Q

The maternity nurse understands that vascular volume increases 40% to 60% during pregnancy to

a. compensate for decreased renal plasma flow.
b. provide adequate perfusion of the placenta.
c. eliminate metabolic wastes of the mother.
d. prevent maternal and fetal dehydration.

A

b. provide adequate perfusion of the placenta.

Rationale: The primary function of increased vascular volume is to transport oxygen and nutrients to the fetus via the placenta.
Renal plasma flow increases during pregnancy.
Assisting with pulling metabolic wastes from the fetus for maternal excretion is one purpose of the increased vascular volume.
However, this answer is not the best because it doesn’t explain the overall purpose and only includes one purpose.
Prevention of dehydration is not the reason for increased vascular volume.

116
Q

A nurse is taking vital signs on a pregnant woman. Preconception pulse was 76 beats/minute. Today the pulse is 97 beats/minute. What action by the nurse is best?

a. Inform the provider immediately.
b. Document findings in the chart.
c. Prepare to start an IV infusion.
d. Retake the pulse in 15 minutes.

A

b. Document findings in the chart.

Rationale: The pulse of a pregnant woman increases about 15 to 20 beats/minute throughout the pregnancy.
The nurse should document the findings, but no other actions are needed as this is a normal finding.

117
Q

Physiologic anemia often occurs during pregnancy as a result of

a. inadequate intake of iron.
b. dilution of hemoglobin concentration.
c. the fetus establishing iron stores.
d. decreased production of erythrocytes.

A

b. dilution of hemoglobin concentration.

Rationale: When blood volume expansion is more pronounced and occurs earlier than the increase in red blood cells, the woman will have physiologic anemia, which is the result of dilution of hemoglobin concentration rather than inadequate hemoglobin.
Inadequate intake of iron may lead to true anemia. If the woman does not take an adequate amount of iron, true anemia may occur when the fetus pulls stored iron from the maternal system.
There is an increased production of erythrocytes during pregnancy.

118
Q

While assessing her patient, what does the nurse interpret as a positive sign of pregnancy?

a. Fetal movement felt by the woman
b. Amenorrhea
c. Breast changes
d. Visualization of fetus by ultrasound

A

d. Visualization of fetus by ultrasound

Rationale: The only positive signs of pregnancy are auscultation of fetal heart tones, visualization of the fetus by ultrasound, and fetal movement felt by the examiner.
Fetal movement felt by the woman, amenorrhea, and breast changes are all presumptive signs.

119
Q

A woman is currently pregnant; she has a 5-year-old son and a 3-year-old daughter born at full term. She had one other pregnancy that terminated at 8 weeks. Her gravida and para are

a. gravida 3 para 2.
b. gravida 4 para 3.
c. gravida 4 para 2.
d. gravida 3 para 3.

A

c. gravida 4 para 2.

Rationale: She has had four pregnancies, including the current one (gravida 4).
She had two pregnancies that terminated after 20 weeks (para 2).
The pregnancy that terminated at 8 weeks is classified as an abortion, which is not included in the gravida-para classification

120
Q

A woman’s last menstrual period was June 10. The nurse estimates the date of delivery (EDD) to be

a. April 7.
b. March 17.
c. March 27.
d. April 17.

A

b. March 17.

Rationale: To determine the EDD, the nurse uses the first day of the last menstrual period (June 10), subtracts 3 months (March 10), and adds 7 days (March 17).
The year is corrected if needed.
April 7 would be subtracting 2 months instead of 3 months and then subtracting 3 days instead of adding 7 days.
March is the correct month, but instead of adding 7 days, 17 days were added to get March 27.
April 17 is subtracting 2 months instead of 3 months.

121
Q

A nurse sees a woman in her first trimester of pregnancy. The nurse explains that the woman can expect to visit her physician every 4 weeks so that

a. she develops trust in the health care team.
b. her questions about labor can be answered.
c. the condition of the mother and fetus can be monitored.
d. problems can be eliminated.

A

c. the condition of the mother and fetus can be monitored.

Rationale: This routine allows monitoring of maternal health and fetal growth and ensures that problems will be identified early.
If the woman begins prenatal care in the first trimester, every 4 weeks is the recommended schedule for visits.
Developing a trusting relationship should be established during these visits, but that is not the primary reason.
Most women do not have questions concerning labor until the last trimester of the pregnancy.
All problems cannot be eliminated because of prenatal visits, but they can be identified.

122
Q

A patient in her first trimester complains of nausea and vomiting. She asks, “Why does this happen?” The nurse’s best response is

a. “It is due to an increase in gastric motility.”
b. “It may be due to changes in hormones.”
c. “It is related to an increase in glucose levels.”
d. “It is caused by a decrease in gastric secretions.”

A

b. “It may be due to changes in hormones.”

Rationale: Nausea and vomiting are believed to be caused by increased levels of hormones and decreased gastric motility. Glucose levels decrease in the first trimester.
Hypoglycemia, if experienced, can also lead to nausea.
Gastric secretions do decrease, but this is not the main cause of nausea and vomiting.

123
Q

The nurse teaches a pregnant woman that one of the most effective methods for preventing venous stasis is to

a. wear elastic stockings in the afternoons.
b. sleep with the foot of the bed elevated.
c. rest often with the feet elevated.
d. sit with the legs crossed.

A

c. rest often with the feet elevated.

Rationale: Elevating the feet and legs improves venous return and prevents venous stasis.
Elastic stockings should be applied before lowering the legs in the morning.
Elevating the legs at night may cause pressure on the diaphragm and increase breathing problems.
Sitting with the legs crossed will decrease circulation in the legs and increase venous stasis.

124
Q

A patient notices that the doctor writes “positive Chadwick’s sign” on her chart. She asks the nurse what this means. The nurse’s best response is

a. “It refers to the bluish color of the cervix in pregnancy.”
b. “It means the cervix is softening.”
c. “The doctor was able to flex the uterus against the cervix.”
d. “That refers to a positive sign of pregnancy.”

A

a. “It refers to the bluish color of the cervix in pregnancy.”

Rationale: Increased vascularity of the pelvic organs during pregnancy results in the bluish color of the cervix, vagina, and labia, called Chadwick’s sign.
The nurse should also know that this is a presumptive, not positive, sign of pregnancy.
Softening of the cervix is Goodell’s sign.
The softening of the lower segment of the uterus (Hegar’s sign) can allow the uterus to be flexed against the cervix.

125
Q

A woman is at her first prenatal visit and is distressed at needing an HIV test. What response by the nurse is best?
a. “We ask all women to be tested for HIV during their pregnancy.”
b. “This test is required by law for pregnant women.”
c. “Infection with HIV will make your pregnancy very high risk.”
d. “You could have been exposed and not know it.”

A

a. “We ask all women to be tested for HIV during their pregnancy.”

Rationale: A voluntary HIV test should be conducted on all women, regardless of risk factors.
This explanation is accurate and helps lessen the woman’s feeling of stigma.
It also lets the woman know it is voluntary. The test is not required by law.
Although an HIV infection will increase the risk of complications, this explanation is too limited to be a good answer.
It is true the woman may have been exposed, but that comment is demeaning and could be offensive.

126
Q

To relieve a leg cramp, the patient should be instructed to

a. massage the affected muscle.
b. stretch and point the toe.
c. dorsiflex the foot.
d. apply a warm pack.

A

c. dorsiflex the foot.

Rationale: Dorsiflexion of the foot stretches the leg muscle and relieves the painful muscle contraction.
Since she is prone to blood clots in the legs, massaging the affected leg muscle is contraindicated.
Pointing the toes will contract the muscle and not relieve the pain.
Warm packs can be used to relax the muscle, but more immediate relief is necessary, such as dorsiflexion of the foot.

127
Q

The multiple marker screen is used to assess the fetus for which condition?

a. Down syndrome
b. Diaphragmatic hernia
c. Congenital cardiac abnormality
d. Anencephaly

A

a. Down syndrome

Rationale: The maternal serum level of alpha-fetoprotein is used to screen for trisomy 18 or 21 and neural tube defects. The quadruple marker test does not detect hernias.
Additional testing, such as ultrasonography, would be required to diagnose diaphragmatic hernia.
Congenital cardiac abnormality would most likely be identified during an ultrasound examination.

128
Q

A nurse is caring for patients in the prenatal clinical who are all 35 weeks along. Which patient should the nurse see first?

a. Shortness of breath when climbing stairs
b. Abdominal pain
c. Ankle edema in the afternoon
d. Backache with prolonged standing

A

b. Abdominal pain

Rationale: Abdominal pain may indicate preterm labor or placental abruption so this patient should be seen first. Shortness of breath climbing stairs, afternoon ankle edema, and backache are all normal findings at this stage of pregnancy.

129
Q

A patient at 32 weeks of gestation reports that she has severe lower back pain. The nurse’s assessment should include

a. observation of posture and body mechanics.
b. palpation of the lumbar spine.
c. exercise pattern and duration.
d. ability to sleep for at least 6 hours uninterrupted.

A

a. observation of posture and body mechanics.

Rationale: Correct posture and body mechanics can reduce lower back pain caused by increasing lordosis.
Pregnancy should not cause alterations in the spine.
Any assessment for malformation should be done early in the pregnancy.
Exercise and sleep are not as important to assess as are posture and body mechanics, which can contribute to the pain.

130
Q

A pregnant couple has formulated a birth plan and is reviewing it with the nurse at an expectant parent’s class. Which aspect of their birth plan would require further discussion with the nurse?

a. “My husband and I have agreed that my sister will be my coach.”
b. “We plan to use Lamaze to reduce the pain during labor.”
c. “We want the labor and birth to take place in a birthing room with our son present.
d. “We will not use the fetal monitor during labor.”

A

d. “We will not use the fetal monitor during labor.”

Rationale: A birth plan consists of what the woman and partner wish to have happen during labor and delivery. Intermittent or continuous fetal monitoring is one aspect of care for consideration; however, it is unrealistic to state that monitoring will not be used.
The nurse should explain the purpose to ensure the couple is making an informed decision.
The woman can refuse this procedure but would need to understand how this might negatively impact her child.
The couple also need to understand that the entire plan is tentative depending on what events actually occur.
The other statements are appropriate for a birth plan.

131
Q

A couple ask the prenatal nurse to explain centering pregnancy. Which statement accurately applies to this model of care?

a. A way to control labor pain and remain centered during the process
b. A philosophy of making the pregnancy the center of the family’s life
c. Education and support sessions are provided to small cohorts of women
d. Labor practice where the woman is surrounded by an extensive network of people

A

c. Education and support sessions are provided to small cohorts of women

Rationale: This method involves ten 1.5- to 2-hour sessions with small groups of women and health care providers beginning at 12 to 16 weeks of pregnancy and ending in early postpartum.
Sessions include assessment, education, and social support.
It is not a way to control labor pain, a philosophy of making the pregnancy the center of the family’s life, or the use of a large network of people during labor.

132
Q

Which comment by a woman in her first trimester indicates ambivalent feelings?

a. “I wanted to become pregnant, but I’m scared about being a mother.”
b. “I haven’t felt well since this pregnancy began.”
c. “I’m concerned about the amount of weight I’ve gained.”
d. “My body is changing so quickly.”

A

a. “I wanted to become pregnant, but I’m scared about being a mother.”

Rationale: Ambivalence refers to conflicting feelings.
This woman is demonstrating this conflict.
The other statements do not indicate ambivalence.

133
Q

A patient who is 7 months pregnant states, “I’m worried that something will happen to my baby.” The nurse’s best response is

a. “There is nothing to worry about.”
b. “The doctor is taking good care of you and your baby.”
c. “Tell me about your concerns.”
d. “Your baby is doing fine.”

A

c. “Tell me about your concerns.”

Rationale: Encouraging the client to discuss her feelings is the best approach.
Women during their third trimester need reassurance that such fears are not unusual in pregnancy.
An open-ended request to share information will encourage the patient to explain concerns further.
The other statements belittle the patient’s concerns and provide false hope.

134
Q

Which of the following behaviors by a pregnant woman would be an example of mimicry?

a. Babysitting for a neighbor’s children
b. Wearing maternity clothes before they are needed
c. Daydreaming about the newborn
d. Imagining oneself as a good mother

A

b. Wearing maternity clothes before they are needed

Rationale: Mimicry refers to observing and copying the behaviors of others, in this case, other pregnant women. Wearing maternity clothes before they are needed helps the expectant mother “feel” what it’s like to be obviously pregnant. Babysitting other children is a form of role playing where the woman practices the expected role of motherhood. Daydreaming is a type of fantasy where the woman “tries on” a variety of behaviors in preparation for motherhood.
Imagining herself as a good mother is the woman’s effort to look for a good role fit.
She observes behavior of other mothers and compares them with her own expectations.

135
Q

A step in maternal role attainment that relates to the woman giving up certain aspects of her previous life is termed

a. looking for a fit.
b. role-playing.
c. fantasy.
d. grief work.

A

d. grief work.

Rationale: The woman experiences sadness as she realizes that she must give up certain aspects of her previous self and that she can never go back.
Looking for a fit is when the woman observes the behaviors of mothers and compares them with her own expectations.
Role playing involves searching for opportunities to provide care for infants in the presence of another person.
Fantasies allow the woman to try on a variety of behaviors. This usually deals with how the child will look and the characteristics of the child.

136
Q

The maternal task that begins in the first trimester and continues throughout the neonatal period is called

a. seeking safe passage for herself and her baby.
b. securing acceptance of the baby by others.
c. learning to give of herself.
d. developing attachment with the baby.

A

d. developing attachment with the baby.

Rationale: Developing attachment (strong ties of affection) to the unborn baby begins in early pregnancy when the woman accepts that she is pregnant.
By the second trimester, the baby becomes real, and feelings of love and attachment surge.
Seeing safe passage is a task that ends with delivery.
During this task the woman seeks health care and cultural practices.
Securing acceptance continues throughout pregnancy as the woman reworks relationships.
Learning to give of herself occurs during pregnancy and is sometimes noticed as the woman gives to others in the form of food or presents

137
Q

Which situation best describes a man “trying on” fathering behaviors?

a. Spending more time with his siblings
b. Taking a nephew to the park to play
c. Reading books on newborn care
d. Exhibiting physical symptoms related to pregnancy

A

b. Taking a nephew to the park to play

Rationale: Interacting with children and assuming the behavior and role of a father best describe a man “trying on” being a father.
The man normally will seek closer ties with his father during this time, not his siblings.
While some fathers do everything they can to learn about infant care, others are not ready to learn when the information is presented, so the nurse should provide the information again after the baby is born and it is more relevant.
Exhibiting symptoms related to pregnancy is called couvade

138
Q

A 36-year-old divorcee with a successful modeling career finds out that her 18-year-old married daughter is expecting her first child. What is a major factor in determining how the woman will respond to becoming a grandmother?

a. Her career
b. Being divorced
c. Her age
d. Age of the daughter

A

c. Her age

Rationale: Age is a major factor in determining the emotional response of prospective grandparents.
Young grandparents may not be happy with the stereotype of grandparents as being old.
Career responsibilities may have demands that make the grandparents not as accessible, but it is not a major factor in determining the woman’s response to becoming a grandmother.
Being divorced is not a major factor that determines adaptation of grandparents.
The age of the daughter is not a major factor that determines adaptation of grandparents.
The age of the grandparent is a major factor.

139
Q

The nurse who practices in a prenatal clinic understands that a major concern of lower socioeconomic groups is to

a. maintain group health insurance on their families.
b. meet health needs as they occur.
c. practice preventive health care.
d. maintain an optimistic view of life.

A

b. meet health needs as they occur.

Rationale: Because of economic uncertainty, lower socioeconomic groups place more emphasis on meeting the needs of the present rather than on future goals.
Lower socioeconomic groups usually do not have group health insurance.
They may value health care but cannot afford preventive health care.
They may struggle for basic needs and often do not see a way to improve their situation.
It is difficult to maintain optimism.

140
Q

What comment by a new mother exhibits understanding of her toddler’s response to a new sibling?

a. “I can’t believe he is sucking his thumb again.”
b. “He is being difficult, and I don’t have time to deal with him.”
c. “My husband will stay with the baby so I can take our son to the park.”
d. “When we brought the baby home, we made our son stop sleeping in the crib.”

A

c. “My husband will stay with the baby so I can take our son to the park.”

Rationale: It is important for a mother to seek time alone with her toddler to reassure him that he is loved.
Toddlers can feel jealous and resentful having to share the mother’s attention.
It is normal for a child to regress when a new sibling is introduced into the home.
As difficult as it is, the mother must make time to spend with the toddler.
Changes in sleeping arrangements should be made several weeks before the birth so that the child does not feel displaced by the new baby.

141
Q

A nurse in labor and delivery is caring for a Muslim woman during the active phase of labor. You note that when you touch her, she quickly draws away. Which response by the nurse is best?

a. Continue to touch her as much as you need to while providing care.
b. Assume that she doesn’t like you and decrease your time with her.
c. Limit touching to a minimum, as this may not be acceptable in her culture.
d. Ask the charge nurse to reassign you to another patient.

A

c. Limit touching to a minimum, as this may not be acceptable in her culture.

Rationale: Touching is an important component of communication in various cultures, but if the patient appears to find it offensive, the nurse should respect her cultural beliefs and limit touching her.
By continuing to touch her, the nurse is showing disrespect for her cultural beliefs.
A cultural response to touch does not reflect like or dislike. Being assigned to another patient is inappropriate; all nurses must be able to provide culturally appropriate care.

142
Q

A nurse is encouraging a patient to attend an early pregnancy class for the second trimester. What topic would be inconsistent with the nurse’s knowledge of topics presented in this class?

a. Fetal development
b. Body mechanics
c. Childbirth choices
d. Managing morning sickness

A

d. Managing morning sickness

Rationale: Managing morning sickness would be taught in a first trimester early pregnancy class.
The other topics are appropriate for second trimester classes.

143
Q

A pregnant patient of 28 weeks’ gestation complains of pain in the right inguinal area. What action by the nurse is best?

a. Assess the woman for early labor.
b. Position the woman on the right side.
c. Palpate the woman’s abdomen.
d. Document the findings in the chart.

A

b. Position the woman on the right side.

Rationale: Pain in the right inguinal area is most likely due to the round ligament.
The nurse can position the woman on her right side to see if that relieves the pain.
Heat can also help.
There is no need to assess for labor or palpate the abdomen.
The findings should be documented after the nurse responds.

144
Q

A student nurse is teaching a pregnant woman ways to manage constipation. Which instruction by the student causes the nurse to provide a correction?

a. Drink at least 8 glasses of liquids a day.
b. The fat in cheese helps lubricate the bowels.
c. You do need to continue your iron pills.
d. Add extra fiber, which can be found in fruit.

A

b. The fat in cheese helps lubricate the bowels.

Rationale: Cheese tends to cause constipation, so this statement by the student needs correction by the nurse.
The other statements are all correct.

145
Q

The nurse is caring for a patient whose English is limited. When the nurse provides information, the patient smiles and nods her head. What action by the patient indicates that the goal for a primary nursing diagnosis for this patient has been met?

a. Keeps next appointment and brings a translator with her.
b. Gains an appropriate amount of weight at next visit.
c. Husband accompanies patient to appointments.
d. Continues to eat culturally appropriate foods.

A

a. Keeps next appointment and brings a translator with her.

Rationale: The primary goal for this situation is Impaired Verbal Communication due to lack of English proficiency.
If the patient is able to understand and keep her next appointment and brings a translator with her to help facilitate communication that shows that the goal of adequate communication has been met.
The other actions do not address communication

146
Q

The nurse in the OB triage area has four patients to see. Which patient should the nurse see first?

a. First trimester, vomiting for an hour
b. Second trimester, fingers swollen
c. Third trimester, painful urination
d. Third trimester, painful vaginal bleeding

A

d. Third trimester, painful vaginal bleeding

Rationale: This patient may have a placenta previa or abruptio placentae or might be having a spontaneous abortion.
The nurse needs to see this patient first.
The other patients may have normal vomiting of the first trimester.
Swollen fingers indicate edema that needs to be investigated. Painful urination probably indicates a urinary tract infection. The priority patient is the one with bleeding.

147
Q

The nurse is caring for a woman who had infibulation performed on her as a child. Which of the following actions by the perinatal nursing staff are appropriate for this patient? (Select all that apply.)

a. Obtaining frequent urinalysis collections
b. Providing larger equipment for exams
c. Astute assessments for pain during procedures
d. Monitoring for infections
e. Draping the woman maximally

A

a. Obtaining frequent urinalysis collections
c. Astute assessments for pain during procedures
d. Monitoring for infections
e. Draping the woman maximally

Rationale: Female genital mutilation, cutting, or circumcision, also called infibulation, involves removal of some or all of the external female genitalia.
The labia majora are often stitched together over the vaginal and urethral opening as part of this practice.
The woman is at risk for many issues including urinary tract and other genital infections and pain.
Often the woman will not give any verbal or nonverbal signs of pain so the nurse must be astute in assessing for it.
Draping the woman should be done as completely as possible. The equipment for exams must be smaller, such as a pediatric speculum.

148
Q

A pregnant woman reports that she works in a long-term care setting and is concerned about the impending flu season. She asks about receiving the flu vaccine. Which vaccines could this patient receive? (Select all that apply.)

a. Tetanus
b. Hepatitis A and B
c. Measles, mumps, rubella (MMR)
d. Influenza
e. Varicella

A

a. Tetanus
b. Hepatitis A and B
d. Influenza

Rationale: Inactivated vaccines such as those for tetanus, hepatitis A, hepatitis B, and influenza are safe to administer for women who have a risk for contracting or developing the disease.
Immunizations with live virus vaccines such as MMR, varicella (chickenpox), or smallpox are contraindicated during pregnancy because of the possible teratogenic effects on the fetus.

149
Q

During pregnancy there are a number of changes that occur as a direct result of the presence of the fetus. Which of these adaptations meet these criteria? (Select all that apply.)

a. Leukorrhea
b. Development of a mucous plug
c. Quickening
d. Ballottement
e. Lightening

A

a. Leukorrhea
c. Quickening
e. Lightening

Rationale: Leukorrhea is a white or slightly gray vaginal discharge that develops in response to cervical stimulation by estrogen and progesterone.
Quickening is the first recognition of fetal movements, or “feeling life.”
Quickening is often described as a flutter and is felt earlier in multiparous women than in primiparas.
Lightening occurs when the fetus begins to descent into the pelvis.
This occurs 2 weeks before labor in the nullipara and at the start of labor in the multipart.
Mucus fills the cervical canal, creating a plug that acts as a barrier against bacterial invasion during pregnancy.
Passive movement of the unengaged fetus is referred to as ballottement

150
Q

A pregnant woman asks the nursing student what to do about her frequent heartburn. What suggestions can the student make that are appropriate? (Select all that apply.)

a. Try chewing gum during the day.
b. Take Alka-Seltzer or other antacid.
c. Drink a small sip of cream before meals.
d. Eat small amounts of dry crackers.
e. Wear loose-fitting clothing.

A

a. Try chewing gum during the day.
c. Drink a small sip of cream before meals.
e. Wear loose-fitting clothing.

Rationale: Chewing gum, a small sip of cream before meals, and wearing loose clothing all can help relieve heartburn.
The patient can take antacids recommended by the provider, but Alka-Seltzer has too much sodium. Dry crackers help with morning sickness.

151
Q
  1. Which pregnant woman should restrict her weight gain during pregnancy?

a. Woman pregnant with twins
b. Woman in early adolescence
c. Woman shorter than 62 inches or 157 cm
d. Woman who was 20 pounds overweight before pregnancy

A

d. Woman who was 20 pounds overweight before pregnancy

Rationale: A higher weight gain in twin gestations may help prevent low birth weights.
B Adolescents need to gain weight toward the higher acceptable range, which will provide for their own growth as well as for fetal growth.
C In the past women of short stature were advised to restrict their weight gain; however, evidence to support these guidelines has not been found.
D A weight gain of 5 to 9 kg will provide sufficient nutrients for the fetus.
Overweight and obese women should be advised to lose weight prior to conception in order to achieve the best pregnancy outcomes.

152
Q

The major source of nutrients in the diet of a pregnant woman should be composed of

a. Simple sugars
b. Fats
c. Fiber
d. Complex carbohydrates

A

d. Complex carbohydrates

Rationale: The most common simple carbohydrate is table sugar, which is a source of energy but does not provide any nutrients.
B Fats provide 9 kcal in each gram, in contrast to carbohydrates and proteins, which provide only 4 kcal in each gram.
C Fiber is supplied mainly by the complex carbohydrates.
D Complex carbohydrates supply the pregnant woman with vitamins, minerals, and fiber.

153
Q

To increase the absorption of iron in a pregnant woman, the iron preparation should be given with

a. Milk
b. Tea
c. Orange juice
d. Coffee

A

c. Orange juice

Rationale: A The calcium and phosphorus in milk decrease iron absorption.
B Tannin in the tea reduces the absorption of iron.
C A vitamin C source may increase the absorption of iron.
D A decreased intake of caffeine is recommended in pregnancy.

154
Q

A pregnant woman’s diet may not meet her need for folates. A good source of this nutrient is

a. Chicken
b. Cheese
c. Potatoes
d. Green leafy vegetables

A

d. Green leafy vegetables

Rationale: Chicken is an excellent source of protein, but it is poor in folates.
B Cheese is an excellent source of calcium, but it is poor in folates.
C Potatoes contain carbohydrates and vitamins and minerals, but are poor in folates.
D Sources of folates include green leafy vegetables, whole grains, fruits, liver, dried peas, and beans.

155
Q

One danger in using nonfood supplementation of nutrients is

a. Increased absorption of all vitamins
b. Development of pregnancy-induced hypertension (PIH)
c. Increased caloric intake
d. Toxic effects on the fetus

A

d. Toxic effects on the fetus

Rationale: Supplements do not have better absorption than natural vitamins and minerals.
B There is no relationship between supplements and PIH.
C Supplements do not contain calories.
D The use of supplements in addition to food may increase the intake of some nutrients to doses much higher than the recommended amounts.
Overdoses of some nutrients have been shown to cause fetal defects.

156
Q

When providing care to the prenatal patient, the nurse understands that pica is defined as

a. Intolerance of milk products
b. Iron deficiency anemia
c. Ingestion of nonfood substances
d. Episodes of anorexia and vomiting

A

c. Ingestion of nonfood substances

Rationale: A This is termed lactose intolerance.
B Pica may produce iron deficiency anemia if proper nutrition is decreased.
C The practice of eating substances not normally thought of as food is called pica.
Clay or dirt and solid laundry starch are the substances most commonly ingested.
D Pica is not related to anorexia and vomiting.

157
Q

The breastfeeding woman whose recommended prepregnant caloric intake was 2000 calories per day needs how many calories per day to meet her current needs?

a. 2300
b. 2500
c. 2750
d. 3000

A

b. 2500

Rationale: A 2300 calories is not enough calories to meet her needs.
B The increase for a breastfeeding mother is 500 calories above her recommended prepregnant caloric intake.
C 2750 calories may be too many calories and may lead to weight gain.
D 3000 calories is too many for this mother and will lead to weight gain.

158
Q

The most important reason for evaluating the pattern of weight gain in pregnancy is to

a. Prevent excessive adipose tissue deposits.
b. Identify potential nutritional problems or complications of pregnancy.
c. Assess the need to limit caloric intake in obese women.
d. Determine cultural influences on the woman’s diet.

A

b. Identify potential nutritional problems or complications of pregnancy.

Rationale: A Excessive adipose tissue may occur with excess weight gain, but it is not the reason for monitoring the weight gain pattern.
B Deviations from the recommended pattern of weight gain may indicate nutritional problems or developing complications.
C It is important to monitor the pattern of weight gain for developing complications.
D The pattern of weight gain is not influenced by cultural influences.

159
Q

If a patient’s normal prepregnancy diet contains 45 g of protein daily, how many more grams of protein should she consume per day during pregnancy?

a. 5
b. 10
c. 26
d. 30

A

c. 26

Rationale: A 5 g will not be enough to meet her protein needs during pregnancy.
B 10 g will not be enough extra protein to meet her needs during pregnancy.
C The recommended intake of protein for the pregnant woman is 71 g.
D 30 g is more than is necessary and will add extra calories.

160
Q

A pregnant patient would like to know a good food source of calcium other than dairy products. Your best answer is

a. Legumes
b. Yellow vegetables
c. Lean meat
d. Whole grains

A

a. Legumes

Rationale: A Although dairy products contain the greatest amount of calcium, it also is found in legumes, nuts, dried fruits, and some dark green leafy vegetables.
B Yellow vegetables are rich in vitamin A.
C Lean meats are rich in protein and phosphorus.
D Whole grains are rich in zinc and magnesium.

161
Q

To determine the cultural influence on a patient’s diet, the nurse should first

a. Evaluate the patient’s weight gain during pregnancy.
b. Assess the socioeconomic status of the patient.
c. Discuss the four food groups with the patient.
d. Identify the food preferences and methods of food preparation common to that culture.

A

d. Identify the food preferences and methods of food preparation common to that culture.

Rationale: A Evaluating a patient’s weight gain during pregnancy should be included for all patients, not just for those who are culturally different.
B The socioeconomic status of the patient may alter the nutritional intake, but not the cultural influence.
C Teaching the food groups to the patient should come after assessing food preferences.
D Understanding the patient’s food preferences and how she prepares food will assist the nurse in determining whether the patient’s culture is adversely affecting her nutritional intake.

162
Q

Which pregnant adolescent is most at risk for a nutritional deficit during pregnancy?

a. A 15-year-old of normal height and weight
b. A 17-year-old who is 10 pounds underweight
c. A 16-year-old who is 10 pounds overweight
d. A 16-year-old of normal height and weight

A

b. A 17-year-old who is 10 pounds underweight

Rationale: A A 15-year-old has special nutritional needs during pregnancy, but she is not at the highest risk for deficiency.
B The adolescent who is pregnant and underweight is most at risk, because she is already deficient in nutrition and must now supply the nutritional intake for both herself and her fetus.
C An overweight pregnant teen is at risk for deficiency, but is not at the highest risk.
Being underweight is the most risky because she is already deficient.
D A 16-year-old has special nutritional needs during pregnancy, but she is not at the highest risk for deficiency.

163
Q

What is a goal of a patient with the following nursing diagnosis: Imbalanced Nutrition: Less Than Body Requirements related to diet choices inadequate to meet nutrient requirements of pregnancy?

a. Gain a total of 30 lb.
b. Take daily supplements consistently.
c. Decrease intake of snack foods.
d. Increase intake of complex carbohydrates.

A

a. Gain a total of 30 lb.

Rationale: A A weight gain of 30 lb is one indication that the patient has gained a sufficient amount for the nutritional needs of pregnancy.
B A daily supplement is not the best goal for this patient. It does not meet the basic need of proper nutrition during pregnancy.
C Decreasing snack foods may be the problem and should be assessed. However, assessing the weight gain is the best method of monitoring nutritional intake for this pregnancy.
D Increasing the intake of complex carbohydrates is important for this patient, but monitoring the weight gain should be the end goal.

164
Q

A patient who is in week 28 of gestation is concerned about her weight gain of 17 lb. The nurse’s best response is

a. “You should try to decrease your amount of weight gain for the next 12 weeks.”
b. “You have gained an appropriate amount for the number of weeks of your pregnancy.”
c. “You should not gain any more weight until you reach the third trimester.”
d. “You have not gained enough weight for the number of weeks of your pregnancy.”

A

b. “You have gained an appropriate amount for the number of weeks of your pregnancy.”

Rationale: A The woman has gained the appropriate amount of weight. It would be inappropriate to have her decrease her weight gain.
B A woman in her 28th week of gestation should have gained between 17 and 20 lb. The normal pattern of weight gain is 2 to 3 lb total in the first trimester (by 13 weeks) and 1 lb per week after that.
C Weight gain needs to be consistent during the last part of the pregnancy and should not be suppressed.
D She has gained an appropriate amount of weight and should not increase the weight gain.

165
Q

In teaching the pregnant adolescent about nutrition, the nurse should

a. Emphasize the need to eliminate common teen snack foods, because they are too high in fat and sodium.
b. Determine the weight gain needed to meet adolescent growth and add 35 lb.
c. Suggest that she not eat at fast-food restaurants, to avoid foods of poor nutritional value.
d. Realize that most adolescents are unwilling to make dietary changes during pregnancy.

A

b. Determine the weight gain needed to meet adolescent growth and add 35 lb.

Rationale: Changes in the diet should be kept at a minimum, and snacks should be included.
Snack foods can be in included in moderation and other foods added to make up for the lost nutrients.
B Adolescents should gain in the upper range of the recommended weight gain.
They also need to gain weight that would be expected for their own normal growth.
C Eliminating fast foods will make her appear different to her peers.
She should be taught to choose foods that add needed nutrients.
D Adolescents are willing to make changes; however, they still have the need to be like their peers.

166
Q

The recommended diet for pregnancy differs from the recommended diet for lactation, because

a. Lactating women require more calories and protein.
b. Pregnant women need more calcium.
c. Lactating women require fewer vitamins.
d. Pregnant women require more iron and protein.

A

a. Lactating women require more calories and protein.

Rationale: A The lactating woman needs 200 calories and 5 g of protein more than the pregnant woman.
B Calcium needs are the same for pregnancy and lactation.
C Vitamin needs are higher during pregnancy and lactation.
D Protein requirements are higher during lactation.

167
Q

The traditional diet of Asian women includes little meat or dairy products and may be low in calcium and iron. The nurse can help the woman increase her intake of these foods by

a. Emphasizing the need for increased milk intake during pregnancy
b. Suggesting she eat more “hot” foods during pregnancy
c. Telling her husband that she must increase her intake of fruits and vegetables for the baby’s sake
d. Suggesting she eat more tofu, bok choy, and broccoli

A

d. Suggesting she eat more tofu, bok choy, and broccoli

Rationale: A Since milk products are not part of the of this woman’s diet, it should be respected and other alternatives offered.
Also, lactose intolerance is common.
B Pregnancy is considered “hot”; therefore the woman would eat “cold” foods.
C Fruits and vegetables are “cold” foods and included in the diet. In family dynamics, however, the husband does not dictate to the wife in this culture.
D The diet should be improved by increasing foods acceptable to the woman. These foods are common in the Asian diet and are good sources of calcium and iron.

168
Q

A pregnant woman’s diet consists almost entirely of whole grain breads and cereals, fruits, and vegetables. The nurse should be most concerned about this woman’s intake of

a. Calcium
b. Protein
c. Vitamin B12
d. Folic acid

A

c. Vitamin B12

Rationale: A Depending upon the woman’s food choices this diet may be adequate in calcium.
B Protein needs can be sufficiently met by a vegetarian diet.
C This diet is consistent with that followed by a strict vegetarian (vegan). Vegans consume only plant products. Because vitamin B12 is found in foods of animal origin, this diet is deficient in vitamin B12.
D The nurse should be more concerned with the woman’s intake of vitamin B12 due to her dietary restrictions. Folic acid needs can be met by enriched bread products.

169
Q

Which statement made by a lactating woman leads the nurse to believe that the woman might have lactose intolerance?

a. “I always have heartburn after I drink milk.”
b. “If I drink more than a cup of milk, I usually have abdominal cramps and bloating.”
c. “Drinking milk usually makes me break out in hives.”
d. “Sometimes I notice that I have bad breath after I drink a cup of milk.”

A

b. “If I drink more than a cup of milk, I usually have abdominal cramps and bloating.”

Rationale: A The woman with lactose intolerance is more likely to experience bloating and cramping, not heartburn.
B One problem that can interfere with milk consumption is lactose intolerance, which is the inability to digest milk sugar because of a lack of the enzyme lactose in the small intestine. Milk consumption may cause abdominal cramping, bloating, and diarrhea in such people, although many lactose-intolerant individuals can tolerate small amounts of milk without symptoms.
C A woman who breaks out in hives after consuming milk is more likely to have a milk allergy.
D This woman should be advise to simply brush her teeth after consuming dairy products.

170
Q

To prevent GI upset, patients should be instructed to take iron supplements

a. On a full stomach
b. At bedtime
c. After eating a meal
d. With milk

A

b. At bedtime

Rationale: A Iron supplements are best absorbed if they are taken when the stomach is empty.
B Taking iron supplements at bedtime may reduce GI upset.
C Iron supplements are best absorbed if they are taken when the stomach is empty.
Iron can be taken at bedtime if abdominal discomfort occurs when it is taken between meals.
D Bran, tea, coffee, milk, and eggs may reduce absorption. Iron can be taken at bedtime if abdominal discomfort occurs when it is taken between meals.

171
Q

After you complete your nutritional counseling for a pregnant woman, you ask her to repeat your instructions so that you can assess her understanding of the instructions given. Which statement indicates that she understands the role of protein in her pregnancy?

a. “Protein will help my baby grow.”
b. “Eating protein will prevent me from becoming anemic.”
c. “Eating protein will make my baby have strong teeth after he is born.”
d. “Eating protein will prevent me from being diabetic.”

A

a. “Protein will help my baby grow.”

Rationale: A Protein is the nutritional element basic to growth. An adequate protein intake is essential to meeting the increasing demands of pregnancy.
These demands arise from the rapid growth of the fetus; the enlargement of the uterus, mammary glands, and placenta; the increase in the maternal blood volume; and the formation of amniotic fluid.
B Iron intake prevents anemia.
C Calcium intake is needed for fetal bone and tooth development.
D Glycemic control is needed in diabetics; protein is one nutritional factor to consider, but this is not the primary role of protein intake.

172
Q

Which nutritional recommendation about fluids is accurate?

a. A woman’s daily intake should be 8-10 cups
b. Coffee should be limited to no more than 2 cups, but tea and cocoa can be consumed without worry.
c. Of the artificial sweeteners, only aspartame has not been associated with any maternity health concerns.
d. Water with fluoride is especially encouraged because it reduces the child’s risk of tooth decay.

A

a. A woman’s daily intake should be 8-10 cups

Rationale: A 8-10 cups is the standard for fluids; however, they should be the right fluids.
B All beverages containing caffeine, including tea, cocoa, and some soft drinks, should be avoided or should be drunk only in limited amounts.
C Artificial sweeteners, including aspartame, have no ill effects on the normal mother or fetus.
However, mothers with phenylketonuria (PKU) should avoid aspartame.
D No evidence indicates that prenatal fluoride consumption reduces childhood tooth decay. However, it still helps the mother.

173
Q

Which vitamins or minerals can lead to congenital malformations of the fetus if taken in excess by the mother?

a. Zinc
b. Vitamin D
c. Folic acid
d. Vitamin A

A

b. Vitamin D

Rationale: A Zinc is vital to good maternity and fetal health and is highly unlikely to be consumed in excess. Vitamin A, taken in excess, causes a number of problems. An analog of vitamin A appears in prescribed acne medications, which must not be taken during pregnancy.
B Vitamin D is vital to good maternity and fetal health and is highly unlikely to be consumed in excess.
Vitamin A, taken in excess, causes a number of problems. An analog of vitamin A appears in prescribed acne medications, which must not be taken during pregnancy.
C Folic acid is vital to good maternity and fetal health and is highly unlikely to be consumed in excess.
Vitamin A, taken in excess, causes a number of problems. An analog of vitamin A appears in prescribed acne medications, which must not be taken during pregnancy.
D Zinc, vitamin D, and folic acid are vital to good maternity and fetal health and are highly unlikely to be consumed in excess. Vitamin A, taken in excess, causes a number of problems.
An analog of vitamin A appears in prescribed acne medications, which must not be taken during pregnancy.

174
Q

Eating disorders include anorexia nervosa and bulimia. Many women with anorexia have amenorrhea and do not become pregnant where as women with bulimia or subclinical anorexia may become pregnant. These condition conditions are associated with (select all that apply)

a. Food cravings
b. Low birth weight
c. Food aversions
d. Electrolyte imbalance
e. Small for gestational age infants

A

b. Low birth weight
d. Electrolyte imbalance
e. Small for gestational age infants

Rationale: These conditions are associated with electrolyte imbalance, low birth weight, and small for gestational age infants.
All women should be asked about eating disorders and nurses should watch for behaviors that may indicate disordered eating.
Some women eat normally during pregnancy for the sake of the fetus, but others continue their previous dysfunctional eating patterns during pregnancy or in the early postpartum period.

175
Q

A pregnant woman’s biophysical profile score is 8. She asks the nurse to explain the results. The nurse’s best response is

a. “The test results are within normal limits.”
b. “Immediate delivery by cesarean birth is needed.”
c. “Further tests are needed to determine the meaning of this score.”
d. “We will inform you of your options within the next week.”

A

a. “The test results are within normal limits.”

Rationale: The normal biophysical score ranges from 8 to 10 points if the amniotic fluid volume is adequate.
A normal score allows conservative treatment of high-risk patients.
Delivery can be delayed if fetal well-being is indicated.
An immediate delivery is not needed.
The results of the biophysical profile are usually available immediately after the procedure is performed.

176
Q

Which analysis of maternal serum may predict chromosomal abnormalities in the fetus?

a. Multiple-marker screening
b. Lecithin/sphingomyelin (L/S) ratio
c. Biophysical profile
d. Type and crossmatch of maternal and fetal serum

A

a. Multiple-marker screening

Rationale: Maternal serum can be analyzed for abnormal levels of alpha-fetoprotein, human chorionic gonadotropin, and estriol.
This multiple-marker screening may predict chromosomal defects in the fetus.
The L/S ratio is used to determine fetal lung maturity.
A biophysical profile is used for evaluating fetal status during the antepartum period.
Five variables are used, but none is concerned with chromosomal problems.
The blood type and crossmatch will not predict chromosomal defects in the fetus.

177
Q

The nurse providing care for the pregnant woman understands that a factor indicating the need for fetal diagnostic procedures is

a. maternal diabetes.
b. maternal age older than 30 years.
c. previous infant more than 3000 g at birth.
d. weight gain of 25 pounds.

A

a. maternal diabetes.

Rationale: Diabetes is a risk factor in pregnancy because of possible impairment of placental perfusion.
Other indications for testing include a maternal age greater than 35 years, having had another infant weighing greater than 4000 g at birth, or excessive weight gain.
A weight gain of 25 to 35 pounds is recommended for the woman who begins pregnancy at a normal weight.

178
Q

When is the best time to determine gestational age based on biparietal diameter through ultrasound?

a. First trimester only
b. Second trimester only
c. Any time
d. Second half of pregnancy

A

d. Second half of pregnancy

Rationale: The biparietal diameter is used to determine gestational age during the second half of pregnancy.

179
Q

The primary reason for evaluating alpha-fetoprotein (AFP) levels in maternal serum is to determine if the fetus has

a. hemophilia.
b. a neural tube defect.
c. sickle cell anemia.
d. a normal lecithin/sphingomyelin (L/S) ratio.

A

b. a neural tube defect.

Rationale: An open neural tube allows a high level of AFP to seep into the amniotic fluid and enter the maternal serum. Hemophilia is a genetic defect and is best detected with chromosomal studies such as chorionic villus sampling or amniocentesis.
Sickle cell is a genetic defect and is best detected with chromosomal studies such as chorionic villus sampling or amniocentesis.
L/S ratios are determined with an amniocentesis, which is usually done in the third trimester.

180
Q

While working with the pregnant woman in her first trimester, the nurse is aware that chorionic villus sampling (CVS) can be performed during pregnancy as early as _____ weeks.

a. 4
b. 8
c. 10
d. 12

A

c. 10

Rationale: CVS is usually performed between 10 and 13 weeks of gestation to diagnose fetal chromosomal, metabolic, or DNA abnormalities.

181
Q

The nurse’s role in diagnostic testing is to provide

a. advice to the couple.
b. assistance with decision making.
c. information about the tests.
d. reassurance about fetal safety.

A

c. information about the tests.

Rationale: The nurse should provide the couple with all necessary information about a procedure so that the couple can make an informed decision.
The nurse’s role is to inform, not to advise the couple.
Decision-making should always lie with the couple involved. Ensuring fetal safety is not possible with all of the diagnostic testing.
To offer this is to give false reassurance to the parents.

182
Q

The nurse teaches a pregnant woman that which diagnostic test evaluates the effect of fetal movement on fetal heart activity?

a. Contraction stress test (CST)
b. Sonography
c. Biophysical profile
d. Nonstress test (NST)

A

d. Nonstress test (NST)

Rationale: An NST evaluates the ability of the fetal heart to accelerate either spontaneously or in association with fetal movement.
CST evaluates the fetal reaction to contractions.
Sonographic examinations visualize the fetus and are done for various other reasons.
The biophysical profile evaluates fetal status using many variables.

183
Q

Which nursing intervention is necessary before a second trimester transabdominal ultrasound?

a. Place the woman NPO for 12 hours.
b. Instruct the woman to drink 1 to 2 quarts of water.
c. Administer a soapsuds enema.
d. Perform an abdominal prep.

A

b. Instruct the woman to drink 1 to 2 quarts of water.

Rationale: During the second trimester, a full bladder may be needed to displace the intestines and elevate the uterus for better visibility.
If indicated, the woman should be instructed to drink several glasses of clear fluid an hour before the time of the examination and to delay urination until the examination is completed.
Since she needs to fill her bladder, being NPO is not appropriate.
Enemas and abdominal preps are not necessary for this procedure.

184
Q

The major advantage of chorionic villus sampling (CVS) over amniocentesis is that it

a. is not an invasive procedure.
b. does not require hospitalization.
c. has less risk of spontaneous abortion.
d. is performed earlier in pregnancy.

A

d. is performed earlier in pregnancy.

Rationale: CVS is performed between 10 and 13 weeks of gestation, providing earlier results than amniocentesis, which is normally done during the second and third trimesters, although it can be done as early as 11 weeks if needed.
The woman does not need hospitalization for this invasive procedure, and the risk of spontaneous abortion is about the same for both procedures.

185
Q

What is the purpose of amniocentesis for the patient hospitalized at 34 weeks with pregnancy-induced hypertension?

a. Identification of abnormal fetal cells
b. Detection of metabolic disorders
c. Determination of fetal lung maturity
d. Identification of sex of the fetus

A

c. Determination of fetal lung maturity

Rationale: During the third trimester, amniocentesis is most often performed to determine fetal lung maturity.
In pregnancy-induced hypertension, preterm delivery may be necessary because of changes in placental perfusion.
It is not done to identify abnormal fetal cells, detect metabolic disorders, or identify the sex of the fetus.

186
Q

An NST in which two or more fetal heart rate (FHR) accelerations of 15 beats per minute (bpm) or more occur with fetal movement in a 20-minute period is termed

a. nonreactive.
b. positive.
c. negative.
d. reactive.

A

d. reactive.

Rationale: The NST is reactive (normal) when two or more FHR accelerations of at least 15 bpm (each with a duration of at least 15 seconds) occur in a 20-minute period.
A nonreactive result means that the heart rate did not accelerate during fetal movement.
Positive and negative are not results given with this test.

187
Q

The purpose of initiating contractions in a CST is to

a. determine the degree of fetal activity.
b. apply a stressful stimulus to the fetus.
c. identifying fetal acceleration patterns.
d. increase placental blood flow.

A

b. apply a stressful stimulus to the fetus.

Rationale: The CST involves recording the response of the FHR to stress induced by uterine contractions.
The NST and biophysical profiles look at fetal movements.
The NST looks at fetal heart accelerations with fetal movements.
The CST records the fetal response to stress. It does not increase placental blood flow.

188
Q

A pregnant woman states “This test isn’t my idea, but my husband insists.” Which response by the nurse is most appropriate?

a. “Don’t worry. Everything will be fine.”
b. “Why don’t you want to have this test?”
c. “You’re concerned about having this test?”
d. “It’s your decision.”

A

c. “You’re concerned about having this test?”

Rationale: The nurse should clarify the statement and assist the patient in exploring her feelings about the test.
Stating that everything will be fine is giving false reassurance and belittles the woman’s concerns.
“Why” questions usually put people on the defensive and are not therapeutic.
Of course having the test is the woman’s decision, but this closed statement does not encourage the woman to express her feelings.

189
Q

A nurse is preparing a woman for a nonstress test (NST). What nursing action is most appropriate?

a. Position the woman on her left side.
b. Seat the woman comfortably in a recliner.
c. Have the woman to drink 1 liter of water prior to the test.
d. Place conduction gel on the obese woman’s abdomen.

A

b. Seat the woman comfortably in a recliner.

Rationale: To correctly position the pregnant patient for an NST, the woman usually sits in a reclining chair.
Alternatively she can be in a semi-Fowler position with a lateral tilt.
This will optimize uterine perfusion and prevent supine hypotension.
The woman does not need to drink water. Conduction gel is used in all NST tests.

190
Q

Which statement regarding various biochemical assessments used during pregnancy is correct?

a. Chorionic villus sampling (CVS) is becoming more popular because it provides early diagnosis.
b. Screening for maternal serum alpha-fetoprotein (MSAFP) levels is recommended between 10 and 12 weeks of gestation in order to give parents time to consider options.
c. Percutaneous umbilical blood sampling (PUBS) is one of the multiple marker screen tests for Down syndrome.
d. MSAFP is a screening tool only; it identifies candidates for more definitive procedures.

A

d. MSAFP is a screening tool only; it identifies candidates for more definitive procedures.

Rationale: MSAFP is a screening tool, not a diagnostic tool. Further diagnostic testing is indicated after an abnormal MSAFP.
CVS does provide a rapid result, but it is declining in popularity because of advances in noninvasive screening techniques. MSAFP screening is recommended for all pregnant women. Screening is recommended between 15 and 20 weeks of gestation.
Abnormal findings give parents time to have additional tests done.

191
Q

A woman is scheduled for an ultrasound and is asking the nurse questions about this test. Which statement by the nurse regarding ultrasonography during pregnancy is most accurate?

a. Ultrasonography uses infrared technology to create an image.
b. Ultrasonography is only utilized as an adjunct to more invasive tests.
c. Ultrasonography is not harmful to the fetus.
d. Ultrasonography is not a component of biophysical profile testing.

A

c. Ultrasonography is not harmful to the fetus.

Rationale: Most women look forward to the results of this test, which causes no harm to the fetus.
Ultrasonography uses sound waves to create an image.
As an adjunct to more invasive tests, ultrasonography can provide visual guidance for increased safety.
It can be done as a standalone test.
Ultrasonography is a component of biophysical profile testing.

192
Q

A nurse is teaching a woman how to do “kick counts.” What information about this assessment is most appropriate?

a. Notify your provider if the baby’s movement patterns change.
b. Count the number of fetal movements over 2 hours.
c. Call the OB triage area if there are fewer than 10 movements/hour.
d. Have your partner verify your count at the same time you perform it.

A

a. Notify your provider if the baby’s movement patterns change.

Rationale: Since there is no consensus on how the mother should be taught to perform this assessment, it is more important that she become familiar with her baby’s movements and patterns and notify the provider about any change from normal.

193
Q

A patient is at 6 weeks’ gestation and is having a transvaginal ultrasound. While preparing the patient for this procedure, she expresses concerns over the necessity for this test. The nurse explains that this diagnostic test may be necessary to determine which of the following? (Select all that apply.)

a. Multifetal gestation
b. Bicornuate uterus
c. Presence and location of pregnancy (intrauterine or elsewhere)
d. Amniotic fluid volume
e. Presence of ovarian cysts

A

a. Multifetal gestation
b. Bicornuate uterus
c. Presence and location of pregnancy (intrauterine or elsewhere)
e. Presence of ovarian cysts

Rationale: A transvaginal ultrasound done in the first trimester can detect multifetal gestations, bicornuate uterus, presence and location of pregnancy, and presence of ovarian cysts. Amniotic fluid volume is assessed during the second and third trimesters.

194
Q

The nurse teaches a student that indications for percutaneous umbilical cord sampling (PUBS) include which of the following? (Select all that apply.)

a. Rh disease
b. Fetal well-being
c. Infection
d. Lung maturity
e. Karyotyping

A

a. Rh disease
c. Infection
e. Karyotyping

Rationale: Rh disease, infection, and, infrequently, for karyotyping are all indications for PUBS.
NST or BPP are used to determination fetal well-being. An amniocentesis is done in order to determine lung maturity.

195
Q

The nurse caring for a woman hospitalized for hyperemesis gravidarum should expect that initial treatment involves

a. corticosteroids to reduce inflammation.
b. IV therapy to correct fluid and electrolyte imbalances.
c. an antiemetic, such as pyridoxine, to control nausea and vomiting.
d. enteral nutrition to correct nutritional deficits.

A

b. IV therapy to correct fluid and electrolyte imbalances.

Rationale: Initially, the woman who is unable to down clear liquids by mouth requires IV therapy for correction of fluid and electrolyte imbalances.
Corticosteroids are not the expected treatment for this disorder.
Pyridoxine is vitamin B6, not an antiemetic.
Promethazine, a common antiemetic, may be prescribed.
In severe cases of hyperemesis gravidarum, enteral nutrition via a feeding tube may be necessary to correct maternal nutritional deprivation.
This is not an initial treatment for this patient.

196
Q

A physician prescribes clomiphene citrate (Clomid, Serophene) for a woman experiencing infertility. She is very concerned about the risk of multiple births. Which response by the nurse is most appropriate?

a. “This is a legitimate concern. Would you like to discuss this further before your treatment begins?”
b. “No one has ever had more than triplets with Clomid.”
c. “Ovulation will be monitored with ultrasound so that this will not happen.”
d. “That has a very low chance of happening, so you don’t need to worry too much.”

A

a. “This is a legitimate concern. Would you like to discuss this further before your treatment begins?”

Rationale: The incidence of multiple pregnancies with the use of these medications is increased.
The patient’s concern is legitimate and should be discussed so that she can make an informed decision.
Women have had more that triplets on this medication. Ultrasound cannot prevent multiple gestation.
Telling the woman not to worry is belittling her concerns.

197
Q

A couple comes in for an infertility workup, having attempted to get pregnant for 2 years. The woman, 37, has always had irregular menstrual cycles but is otherwise healthy. The man has fathered two children from a previous marriage and had a vasectomy reversal 2 years ago. The man has had two normal semen analyses, but the sperm seem to be clumped together. What additional test is needed?

a. Testicular biopsy
b. Antisperm antibodies
c. FSH level
d. Examination for testicular infection

A

c. FSH level

Rationale: The woman has irregular menstrual cycles.
The scenario does not indicate that she has had any testing related to this irregularity.
Hormone analysis is performed to assess endocrine function of the hypothalamic-pituitary-ovarian axis when menstrual cycles are absent or irregular.
Determination of blood levels of prolactin, FSH, luteinizing hormone (LH), estradiol, progesterone, and thyroid hormones may be necessary to diagnose the cause of irregular menstrual cycles.
A testicular biopsy would be indicated only in cases of azoospermia (no sperm cells) or severe oligospermia (low number of sperm cells).
Antisperm antibodies are produced by a man against his own sperm.
This is unlikely to be the case here, because the husband has already produced children.
Examination for testicular infection should be done before semen analysis.
Furthermore, infection affects spermatogenesis. However, the woman’s hormone levels would likely be tested first

198
Q

A couple is trying to cope with an infertility problem. They want to know what they can do to preserve their emotional equilibrium. What response by the nurse is most appropriate?

a. “Tell your friends and family so that they can help you.”
b. “Talk only to other friends who are infertile, because only they can help.”
c. “Get involved with a support group. I’ll give you some names.”
d. “You might start thinking about adoption to end this roller coaster of emotion.”

A

c. “Get involved with a support group. I’ll give you some names.”

Rationale: Venting negative feelings may unburden the couple. A support group may provide a safe haven for the couple to share their experiences and gain insight from others’ experiences.
Although talking about their feelings may unburden them of negative feelings, infertility can be a major stressor that affects the couple’s relationships with family and friends who often don’t understand the couple’s feelings.
It is not reasonable to suggest they only talk to other infertile couples.
The nurse should not suggest the couple consider adoption while they are still trying to conceive, plus adoption has its own set of stressors.

199
Q

A couple are asking the nurse about in vitro fertilization. What explanation by the nurse is best?

a. “IVF places the product of conception from your sperm and her egg into the uterus.”
b. “A donor embryo will be transferred into your wife’s uterus.”
c. “Donor sperm will be used to inseminate your wife.”
d. “Don’t worry about the technical stuff; that’s what we are here for.”

A

a. “IVF places the product of conception from your sperm and her egg into the uterus.”

Rationale: A woman’s eggs are collected from her ovaries, fertilized in the laboratory with sperm, and transferred to her uterus after normal embryonic development has occurred. There are no donors involved in this specific type of assisted reproductive technology although if the process does not work due to problems with either the man or the woman, donor products can be used.
Telling the couple to not worry about the technical aspects of the treatment does not offer them any information and belittles their questions and concerns.

200
Q

With regard to the assessment of female, male, and couple infertility, nurses should be aware of which of the following?

a. The couple’s religious, cultural, and ethnic backgrounds do not affect the diagnosis.
b. The investigation is lengthy and can be very costly.
c. The woman is assessed first; if she is not the problem, the male partner is analyzed.
d. Semen analysis is for men; the postcoital test is for women.

A

b. The investigation is lengthy and can be very costly.

Rationale: Fertility assessment and diagnosis take time, money, and commitment from the couple.
Religious, cultural, and ethnic-bred attitudes about fertility and related issues always have an impact on diagnosis and assessment.
Both partners are assessed systematically and simultaneously, as individuals and as a couple.
Semen analysis is for men, but the postcoital test is for the couple.

201
Q

A woman has been prescribed metformin at the infertility clinic. She says “Why am I on this? I am not a diabetic; my sister takes it for her diabetes!” What response by the nurse is best?

a. “It is used to promote ovulation in polycystic ovary disease.”
b. “It will prevent your body from forming antibodies to sperm.”
c. “It helps prepare the uterine lining for eventual implantation.”
d. “I don’t know but I will find out and let you know right away.”

A

a. “It is used to promote ovulation in polycystic ovary disease.”

Rationale: Metformin is used as an adjunctive therapy to promote ovulation in the woman with polycystic ovary disease. It does not prevent antibody formation or prepare the uterine lining.
The nurse should know this information but if he or she does not know, finding out and telling the woman as soon as possible would be the correct response.

202
Q

A nurse is teaching a couple about basal body temperature. What information is most accurate?

a. Measures the man’s scrotal temperature related to sperm production.
b. Basal body temperature is the average resting temperature in the woman.
c. It detects slight temperature elevation just prior to ovulation in the woman
d. Ovulation is the only event that affects the change in body temperature.

A

c. It detects slight temperature elevation just prior to ovulation in the woman

Rationale: This method assesses for the slight rise in temperature just prior to ovulation. It is done on the woman and not the man.
Other factors such as illness and poor sleep can affect the reading.

203
Q

The nurse is reviewing the educational packet provided to a patient about tubal ligation. What important facts should the nurse point out? (Select all that apply.)

a. “It is highly unlikely that you will become pregnant after the procedure.”
b. “This is an effective form of 100% permanent sterilization.”
c. “Sterilization offers protection against sexually transmitted diseases.”
d. “Sterilization offers no protection against sexually transmitted diseases.”
e. “Your menstrual cycle will greatly increase after your sterilization.”

A

a. “It is highly unlikely that you will become pregnant after the procedure.”
d. “Sterilization offers no protection against sexually transmitted diseases.”

Rationale: A woman is unlikely to get pregnant after a tubal ligation, but it is not impossible.
Sterilization does not offer protection against STDs.
Typically, the menstrual cycle remains the same after a tubal ligation.